Download NEET PG 2017 Question Paper with Answers

Download NEET PG (The National Eligibility cum Entrance Test Postgraduate) 2017 Question Paper with Answers





Anatomy
Question 1
Identify the type of muscle shown in the image below.
A> Cruciate
B> Multipennate
C> Parallel
D> Unipennate
Answer - B. Multipennate
Explanation -
Pectoralis Major has multiple rows of diagonal fibres with a single tendon, Hence is a multipennate muscle.
Question 2
Nerves of Branchial arch derived from:
A> Mesoderm
B> Endoderm
C> Neural crest
D> Neuroectoderm
Answer - C
Explanation - Branchial or pharyngeal arches are masses of mesoderm covered by ectoderm and lined by
endoderm. Within these masses, muscular and skeletal components develop, as well as aortic arches and
nerve networks. The arches are separated by grooves, visible on the surface of the embryo as pharyngeal
clefts and in the interior as the pharyngeal pouches
In the human embryo, the arches are first seen during the 4th week of development.
They appear as a series of outpouchings of mesoderm on both sides of developing pharynx.


The neural crest are bilaterally paired strips of cells arising in the ectoderm at the margins of the neural
tube. These cells migrate to many different locations and differentiate into many cell types within the
embryo.
Neural Crest Derivatives
A key feature of neural crest is the migration into other embryonic tissues to form specific neural and non-
neural populations and structures.
Cranial neural crest
migration - dorsolaterally and into pharyngeal arches
craniofacial mesenchyme - cartilage, bone, cranial neurons, glia, and connective tissues of the face
pharyngeal arches and pouches - thymic cells, tooth odontoblasts, middle ear bones (ossicles), stria
vascularis cells, and jaw (mandible)
In the body region, neural crest cells also contribute the peripheral nervous system (both neurons and glia)
consisting of sensory ganglia (dorsal root ganglia), sympathetic and parasympathetic ganglia and neural
plexuses within specific tissues/organs.
In the head region, neural crest cells migrate into the pharyngeal arches forming ectomesenchyme
contributing tissues which in the body region are typically derived from mesoderm (cartilage, bone, and
connective tissue).

Neural Crest Origin
System
Cell Type
PeripheralNervous
Neurons - sensory ganglia, sympathetic and parasympathetic ganglia, enteric
System
nervous system, and plexuses
(PNS)
Neuroglial cells, olfactory ensheathing cells
Schwann cells
Endocrine
Adrenal medulla
Calcitonin-secreting cells
Carotid body type I cells
Integumentary
Epidermal pigment cells
Facial cartilage and Facial and anterior ventral skull cartilage and bones
bone
Sensory
Inner ear, corneal endothelium and stroma
Connective tissue
Tooth papillae
smooth muscle, and adipose tissue of skin of head and neck
Connective tissue of meninges, salivary, lacrimal, thymus, thyroid, and pituitary
glands
Connective tissue and smooth muscle in arteries of aortic arch origin


Question 3
Hard palate contains:

A> Keratinised, submucosa, minor salivary gland




B> Keratinised, absent submucosal layer, minor salivary gland
C> Non keratinised, submucosal layer, minor salivary gland
D> Non keratinised, absent submucosa, minor salivary gland
Answer - A
Explanation: The hard palate is located on the roof of the oral cavity, posterior and medial to the alveolar
process of the maxilla.
The bony structure is formed by the palatine processes of the maxilla and the horizontal plates of the
palatine bones.
The periosteum is covered by a firmly attached mucosa centrally, although a submucosa is apparent
laterally containing vessels. The hard palate is continuous with the soft palate posteriorly.
Macroscopic Features
The hard palate is typically a pale pink colour and may have an orange peel appearance from the palatine
salivary glands (more common posteriorly).
Microscopic Features
The hard palate is lined with a keratinising stratified squamous epithelium, tightly bound to the
underlying periosteum of the palatine bone/maxilla. There is minimal submucosa, which becomes more
prominent posteriorly.

Question 4
What is the tensor of vocal cords:

A> Cricothyroid
B> Lateral Cricoarytenoid
C> Thyroarytenoids
D> Posterior cricoarytenoids
Answer- A
Explanation:
Cricothyroid :
Tensor of vocal cords.





Lateral cricoarytenoid:
Abductor of vocal cords.
Thyroarytenoid:
Relaxor of vocal cords.
Posterior cricoarytenoid:
Abductor of vocal cords
Question 5
Ureteric bud develops from:
A> Mesonephros
B> Metanephros
C> Pronephros
D> Genital sinus
Answer- A
Explanation: The ureteric bud, also known as the metanephrogenic diverticulum, is a protrusion from the
mesonephric duct during the development of the urinary and reproductive organs.
It later develops into a conduit (channel) for urine drainage from the kidneys, which, in contrast, originate
from the metanephric blastema.
The metanephrogenic blastema or metanephric blastema (or metanephric mesenchyme, or metanephric
mesoderm) is one of the two embryological structures that give rise to the kidney, the other being the
ureteric bud.

Question 6
About Weber's syndrome which is incorrect:
A> Contralateral hemiplegia
B> Ipsilateral Oculomotor nerve palsy
C> Contralateral Parkinsonism
D> Anterior cerebral peduncle
Answer - D
Explanation:
Weber's syndrome (superior alternating hemiplegia) is a form of stroke characterized by the
presence of an ipsilateral oculomotor nerve palsy and contralateral hemiparesis or hemiplegia.







It is caused by midbrain infarction as a result of occlusion of the paramedianbranches of the
posterior cerebral artery or of basilar bifurcation perforating arteries.
This lesion is usually unilateral and affects several structures in the midbrain:
1. Contralateral parkinsonism because its dopaminergic projections to the basal ganglia innervate
the ipsilateral hemisphere motor field, leading to a movement disorder of the contralateral
body.
2. Contralateral hemiparesis and typical upper motor neuron findings. It is contralateral because
it occurs before the decussation in the medulla.
3. Difficulty with contralateral lower facial muscles and hypoglossal nerve functions.
4. Ipsilateral Oculomotor nerve palsy with a drooping eyelid and fixed wide pupil pointed down
and out. This leads to diplopia.
Question -7
About Sibson's fascia which is incorrect:

A> Attached to the inner border of 2nd rib
B> Covers apical part of lung
C> Part of scalenus anterior muscle
D> Vessel pass above the fascia
Answer-A
Explanation:
The suprapleural membrane is known as Sibson's fascia.
It refers to a thickening of connective tissue that covers the apex of each human lung.
It attaches to the internal border of the first rib and the transverse processes of vertebra C7.


Question - 8
Thrombosis of posterior inferior cerebellar artery causes:








A> Lateral medullary syndrome
B> Weber syndrome
C> Medial medullary syndrome
D> none
ANSWER- A
Explanation:
The posterior inferior cerebellar artery (PICA), the largest branch of the vertebral artery, is one of
the three main arterial blood supplies for the cerebellum, part of the brain.
Occlusion of the posterior inferior cerebellar artery or one of its branches, or of the vertebral artery
leads to lateral medullary syndrome also called Wallenberg syndrome

Question - 9
Broca's area situated in:

A> Inferior frontal gyrus
B> Superior temporal gyrus
C> Angular gyrus
D> None of the above
Answer- A
Explanation:
Broca's area or the Broca area (44) is a region in the frontal lobe of the dominant hemisphere
(usually the left) of the hominid brain with functions linked to speech production.
Inability to speak after injury to the posteriorinferior frontal gyrus of the brain.
Pierre Paul Brocahe identified this region, known as Broca's area.
Difficulty in language production as Broca's aphasia, also called expressive aphasia.
Broca's area is now typically defined in terms of the pars opercularis and pars triangularis of the
inferior frontal gyrus.




BROCA'S AREA(shown in red). Colored region is pars opercularis and pars triangularis of the inferior frontal
gyrus. Broca's area is now typically defined in terms of the pars opercularis and pars triangularis of the
inferior frontal gyrus.

Question - 10
Function of the muscle shown in picture:

A> Flexion
B> Extension
C> Adduction
D> Abduction

Answer- A
Explanation:
Lumbricals are four short hand muscles located in the metacarpus deep to the palmar fascia.
The lumbricals are intrinsic muscles of the hand that flex the metacarpophalangeal joints and
extendthe interphalangeal joints.
The lumbricals are used during an upstroke in writing.

Question - 11
Acute tonsillitis effects which nerve:

A> Glossopharyngeal Nerve
B> Facial Nerve
C> Trigeminal nerve
D> Vagus Nerve
Answer- A







Explanation: Palatine tonsil is supplied by Tonsillar branch of GLOSSOPHARYNGEAL NERVE and maxillary
division of trigeminal nerve but glossopharyngeal is most likely to be damaged during acute tonsillitis and
tonsillectomy.




Question - 12

Structures not passing through Aortic opening:

A> Azygos vein
B> Aorta
C> Thoracic duct
D> Vagus
Answer - D
Explanation:
Aortic opening (Aortic hiatus) is one the three major apertures through the diaphragm & lies at the
level of T12.
A number of structures pass through the aortic hiatus: aorta, azygos vein, thoracic duct, greater
splanchnic nerve.

Question - 13

Newly erupted teeth is covered by:

A> Perikymata
B> Nasmyth's membrane
C> Fibrous tissue

D> all of the above
Answer- B
Explanation:
? Primary enamel cuticle, also called Nasmyth's membrane, is thin membrane of tissue also known as
reduced enamel epithelium (REE) produced by the ameloblast, that covers the tooth once it has
erupted.
? The primary enamel cuticle protects enamel from resorption by cells of the dental sac and also
secretes desmolytic enzymes for elimination of the dental sac, allowing fusion between reduced
enamel epithelium and oral epithelium. This process allows eruption of the tooth without bleeding.

Physiology
Question ? 14
What should be the value of BMI to be considered as "Lethal" in men?

A> 12
B> 18
C> 13
D> 14
Answer: C ? 13
Explanation: BMI Value of 13 is considered as "Lethal" in men.

Body mass index (BMI) is an estimate of total body fat mass
Simply an index of weight for height.
Formula:
Weight in kilograms divided by the square of height in meters.
Body mass index = Weight (kg)/(Height)2 (m)
Uses:
Used to classify and define underweight, overweight & obesity in adults.
Classification & Metrics:
World Health organization categorized BMI values into:
Classification
BMI value
Normal weight
18.5 to 24.9 kg/m2
Underweight
<18.5 kg/m2
Pre-obesity (Pre-OB)
25 to 29.9 kg/m2

Obesity (OB)
> 30 kg/m2
Class I obesity
30.0 - 34.9 kg/m2
Class II obesity
35.0 - 39.9 kg/m2
Class III obesity (Morbid form)
More than 40 kg/m2
BMI value considered lethal for Men is 13
Females withstand and survive even at lower BMI rate upto 11.
Question ? 15
Which receptors are blocked in Myasthenia Gravis?
A> Ach receptors
B> Ca++receptors
C> Na2+ receptors
D> Opioid receptors
Answer: A - Ach receptors
Explanation:
An autoimmune disease
Antibody mediated autoimmune attack of acetylcholine receptors.
Inability of neuromuscular junctions to transmit signals from nerve to muscle
Resulting in muscle weakness and fatigability.
Unresponsive respiratory muscles may cause respiratory failure in severe cases.
Analysis of neuromuscular transmission reveal
Decrease in acetylcholine receptors (AChRs)
Loss of post-junctional folds.
Circulating antibodies to acetylcholine receptors positive.
Diagnosis:
Clinical test: Strength improvement in response to administration of anticholinesterase
agents.
Treatment:
Anti-cholinesterase drugs: Allows accumulation of larger amounts of acetylcholine in
synaptic junctions.
Neostigmine.
Question ? 16
What is the characteristic pattern seen in Brown sequard syndrome

A> C/L loss of joint sense and position
B> C/L loss of pain
C> I/L loss of complete sensory functions
D> C/L motor functions


Answer: Option B - Contralateral loss of pain sensations
Explanation:
If the spinal cord is completely transected ? All the sensations and motor functions distal to
segment of transection are blocked.
Transection of spinal cord on single side results in "Brown Sequard Syndrome"
Functions affected:

Function affected
Position
Description
Motor
Ipsilateral side
Complete loss of motor functions below the
level of transection

Sensory
Combined effects
Some sensory functions are lost on transected
observed
side and others on opposite side.


On the Contralateral side:
Result of disturbance in Spinothalamic
Sensation of pain, cold, &
pathway.
heat
Loss observed 2 to 6 segments below the level

of transection.


Discrete Crude touch
Poorly localized (Still persists).

Partial transmission occurs in opposite

Spinothalamic tract


Question ? 17
When Va / Q is infinity?

A> Partial pressure of O2 becomes zero
B> No exchange of O2 & CO2
C> Partial pressure of CO2 alone becomes zero
D> Partial pressure both CO2 and O2 remain normal
Answer: Option B - No exchange of O2 & CO2
Explanation:
Va represents the ventilation in alveoli.
Q represents the blood flow through the alveolus.
The ratio of Va and Q explains the respiratory exchange, when there is imbalance between alveolar
ventilation and alveolar blood flow.
Ventilation-perfusion ratio is considered normal there is equal amount of blood flow & ventilation
through alveolus.
Va /q ratio is Zero - There is inadequate or nil, but perfusion persists
Va /q ratio is infinity - There is adequate ventilation but no perfusion.
Both in Zero and infinity Va /Q ratio, there is no exchange of gases through the respiratory
membranes of corresponding alveoli.
When Va /Q ratio is infinity,
The alveolar air equilibrates the humidified inspired air
No exchange of oxygen and carbon-di-oxide occurs.

Partial pressures of the O2 and CO2 are 149mmHg & 0 mmHg respectively

Question ? 18
C wave is seen in

A> Iso -volumetric contraction
B> Slow filling at end of diastole
C> End of systole
D> Start of diastole
Answer: Option B - Iso- volumetric contraction
Explanation:
The variations in jugular venous pressure are transmitted to the jugular veins, producing 3 positive
waves (a, c & v) and 2 negative wave(x & y).
C wave:
Produced by bulging of tricuspid valve into right atrium during Iso- volumetric contraction
of right ventricle

Question ? 19
Alpha waves are seen during?

A> Sleep
B> REM movements
C> Relaxed state
D> Active state

Answer: Option C - Relaxed state
Explanation:
Regarded as normal wavefront
Occur when at rest and eyes closed with active/wandering mind ie., associated with decreased
level of attention (either during relaxed/subconscious thinking)
Regular Rhythm (Frequency 8-13 Hz Amplitude - 50-100 V)

Question ? 20
In hypovolemic shock there is -
A> Afferent arteriolar constriction
B> Efferent arteriolar constriction
C> Increased blood flow to kidney
D> Normal cardiac output
Answer: Option A - Afferent arteriole constriction
Explanation:

Inadequate circulation volume.
Poor venous return to heart will decrease the stroke volume & cardiac output.
Compensation by tachycardia & increased systemic vascular resistance (SVR).
Become cold peripherally (shut down).
Most common causes - Fluid loss of any etiology
Hemorrhage
Salt & water loss
Sepsis
Burns

Question ? 21
Components responsible for counter current mechanism in kidney are all except:

A> Sodium outflow in thick ascending limb
B> Water outflow in thin descending limb
C> Sodium outflow in thin ascending limb
D> Flow of tubular fluid from PCT to DCT
Answer: Option C -Sodium outflow in thin ascending limb
Explanation:
Counter-current system occurs in kidney
A system in which inflow runs parallel to, counter to, and in close proximity to the outflow for some
distance.
2 countercurrent mechanisms available:
Countercurrent multiplier at Loop of Henle
Generate high medullary osmotic gradient pressure
Countercurrent exchanger at vasa recta of medullary capillaries
Helps in maintain the medullary osmotic pressure gradient
Substances involved in countercurrent mechanism include:
Sodium actively absorbed with co-transport of potassium & chlorine in thick ascending limb
of loop of Henle.
Water reabsorbed in thin descending limb of loop of Henle.
Urea diffuse out from the medullary collecting ducts into medullary interstitium.

Question ? 22
Glucose is absorbed in intestine by?
A> Secondary active transport
B> Facilitated diffusion
C> Simple diffusion
D> Primary active transport

Answer: Option B - Facilitated diffusion
Explanation:

Facilitated diffusion is a diffusion of large water soluble molecule by a carrier protein.

Glucose and amino acids are transported across the membrane by this method.

Question ? 23
Insulin like growth factor is secreted by:
A> Liver
B>Pituitary gland
C> Pancreas
D> Adrenal glands
Answer:
Option A - Liver
Explanation:

Growth hormone acts on liver to form small proteins called "Somatomedins"
Somatomedins increases bone growth in all aspect
Effects are similar to insulin, hence referred to as "Insulin-like Growth Factor" (IGF).
Four types available -
Most important is Somatomedin C - Specifically referred to as "Insulin - Growth Factor -1 /
IGF - 1"
Binds to carrier protein in blood
Hence, longer duration of action than growth hormone.
Half-life - about 20 Hrs (compared to growth hormone - 20 mins)
Blood concentration of IGF-1, follows the levels of growth hormone.
Growth effects of GH is mostly attributed to somatomedin (rather than its direct effect on bones &
peripheral tissues)

Question ? 24
What is Prosopagnosia?

A> Impairment of consciousness
B> Being unaware of one's problems
C> Difficulty in identifying known faces
D> Failure to identify objects.

Answer: Option C - Difficulty in identifying known faces.
Explanation:
A feature of anxiety disorder
Prosopagnosia is difficulty in identifying known faces
Other features of anxiety disorder include,
Memory impairment without impairment of consciousness
Being unaware of one's problem (Agnosognosia)

Question ? 25
Transport across nucleus is by all except?

A>Caveolins
B>Local signals



C> Importins
D> Rat proteins
Answer: Option D - Rat proteins
Explanation:
Importin - Type of karyopherin
Transporting protein molecules into nucleus by binding to specific recognition sequences,
called Nuclear Localization Sequences (NLS).
2 subunits, importin and importin .
Caveolins:
Family of integral membrane proteins that are principal components of caveolae
membranes
Involved in receptor-independent endocytosis.
Acts as scaffolding proteins within caveolar membranes by compartmentalizing &
concentrating signaling molecules.
Rat proteins:
Recombinant rat IFN- is a bioactive protein intended for use in cell culture applications.

Question ? 26
Iron from enterocytes through

A> Dmt1
B> Dmt2
C> Ferroprotein1

D> GLUT1
Answer: Option A - DMT1
Explanation:
Iron is brought into the cell through an active transport process
Protein DMT-1 (Divalent Metal Transporter-1),
Expressed on the apical surface of enterocytes in initial part of duodenum.
DMT-1 is not specific to iron & transports other substances like zinc, copper, cobalt & manganese.
Biochemistry
Question- 27
Serotonin is also known as?

A> 5-hydroxytryptamine (5-HT)
B>N-methyl phenylamine
C>3-Methoxytyramine
D>Phenethylamine
Answer : A
Explanation:
Serotonin/ 5-hydroxytryptamine (5-HT) is a monoamine neurotransmitter.

Question 28.
Tyrosinosis is caused due to deficiency of which enzyme?

A> Fumarylacetoacetate hydrolase
B>p-hydroxy phenyl pyruvate dehydrogenase
C>Tyrosine transaminase
D>Tyrosine ligase
Answer: A
Explanation: Tyrosinemia, also known as hepatorenal tyrosinemia or tyrosinosis, is the most severe form
of tyrosinemia, a buildup of too much of the amino acid tyrosine in the blood and tissues due to an inability
to metabolize it. It is caused by a deficiency of the enzyme fumarylacetoacetate hydrolase.
Fumarylacetoacetate hydrolase catalyzes the final step in the degradation of tyrosine
Question 29.
Lesch?Nyhan syndrome is caused by deficiency of which enzyme?

A> Orotate Phosphoribosyltransferase
B>Uracil phosphoribosyltransferase
C> Quinolinate Phosphoribosyltransferase
D>Hypoxanthine-guanine phosphoribosyltransferase (HGPRT)
Answer: D
Explanation: Lesch?Nyhan syndrome (LNS), also known as juvenile gout, is a rare inherited disorder caused
by a deficiency of the enzyme hypoxanthine-guanine phosphoribosyltransferase (HGPRT), produced by
mutations in the HPRT gene located on the X chromosome
Question 30.
Fish odour syndrome is caused by deficiency of which enzyme?

A>Fumarylacetoacetate hydrolase
B>Methane monooxygenase
C>Monooxygenase 3 (FMO3)
D> D-amino acid oxidase

Answer: C
Explanation: Trimethylaminuria primary trimethylaminuria), also known as fish odor syndrome or fish
malodor syndrome, is a rare metabolic disorder that causes a defect in the normal production of an
enzyme named flavin-containing monooxygenase 3 (FMO3) When FMO3 is not working correctly or if not
enough enzyme is produced, the body loses the ability to properly convert trimethylamine (TMA) from
precursor compounds in food digestion into trimethylamine oxide (TMAO), through a process called N-
oxidation. Trimethylamine then builds up and is released in the person's sweat, urine, and breath, giving
off a strong fishy odor or strong body odor.
Question 31
Galactosemia is due to deficiency of which enzymes?

A>Galactose-1-phosphate uridyltransferase
B>HGPRT
C>Galactokinase
D>Epimerase

Answer: A
Explanation: Galactosaemia (British galactosaemia) is a rare genetic metabolic disorder that affects an
individual's ability to metabolize the sugar galactose properly. Galactosemia follows an autosomal
recessive mode of inheritance that confers a deficiency in an enzyme responsible for adequate galactose
degradation.
Question 32
Which of the following is most abundant end product of fatty acid synthesis

A>Oleic acid
B>Palmitic acid
C>Arachidonic acid
D>Glucose
Answer: B
Explanation: Fatty acid are synthesized by extramitochondrial system.
This system is present in many tissues including liver kidney brain lung mammary gland and adipose
tissues.
Acetyl CoA is immediate substrate.
The end products of this synthesis are usually the saturated fatty acidspalmitate and stearate with the
latter predominating.
Question 33
About DNA polymerase I which one is correct?

A>Not required in bacteria
B> Repair any damage with DNA

C>Involved in okazaki fragment
D>Participate in DNA replication
Answer: A
Explanation: DNA polymerase I participates in the DNA replication of prokaryotes. Function of Pol I is
mainly to repair any damage with DNA, but it also serves to connect okazaki fragments deleting RNA
primers and replacing the strand with DNA.
Question 34.
What does chaperones assist in?

A>Protein Cleavage
B>Protein Folding
C>Protein Degradation
D>Protein Modification
Answer: B
Explanation: Folding of Proteins in Vivo Is Promoted by Chaperones

Question 35.
Fishy odour occurs due to deficiency of this vitamin from diet

A>Biotin
B>Thiamine
C>Riboflavin
D>Vit. A
Answer: C
Explanation: Vitamin B2 or riboflavin deficiency can bring about a fishy odor in the body
Question 36.
VMA is excreted in urine in which condition

A>Alkaptonuria
B>Phenylketonuria
C>Pheochromocytoma
D>Diabetic ketoacidosis
Answer: C
Explanation: VMA is the end product of catabolism of catecholamines.
In pheochromocytoma and neuroblastoma there is excessive synthesis of catecholamines which causes
enhanced synthesis of VMA and its excretion in the urine.
VMA is the urinary product of both epinephrine and norepinephrine.


It is a good screening test for pheochromocytoma, and is also used to diagnose and follow up
neuroblastoma and ganglioneuroma.
Question 37.
All of the following are excreted in cystinuria except

A>Cystine
B>Cysteine
C>Arginine
D>Orthinithe
Answer: A
Explanation: Cystinuria
? Biochemical Defect :An autosomal recessive disorder that results in the formation of a defective
amino acid transporter in the renal tubule and intestinal epithelial cells.
? Pathophysiology :The amino acid transporter is responsible for transporting cystine, ornithine,
lysine, and arginine . Defective tubular reabsorption of these amino acids in the kidneys results in
increased cysteine in the urine , which can precipitate and cause kidney stones.
? Clinical Manifestations : Cysteine kidney stones presenting with severe, intermittent flank pain and
hematuria.
? Lab findings: Increased urinary excretion of cystine, ornithine, arginine, and lysine on urine amino
acid chromatography; hematuria and cystine crystals (hexagonal) on cooling of acidified urine
sediment.
? Imaging: Radiopaque kidney stones on CT scan.
? Treatment : Low-methionine diet; increased fluid intake; acetazolamide to alkalinize the urine.
Question 38.
Fibrinopeptide A and fibrinopeptide B are acidic due to the presence of which amino acids in its
structure

A>Serine and threonine
B>Glutamate and aspartate
C>Histidine and lysine
D>Glutamine and valine
Answer: B
Explanation: Fibrinopeptides A and B (FPA and FPB) are short amino acid sequences situated at the amino
terminal of the alpha and beta chains (respectively) of soluble fibrinogen. Glutamate and aspartate are
acidic amino acid present in them which turn them acidic.


Question 39
HIAA in urine present in?

A>Alkaptonuria
B>Albinism
C>Carcinoid
D>Phenylketonuria
Answer: C
Explanation: Carcinoid syndrome develops in some people with carcinoid tumors and is characterized by
cutaneous flushing, abdominal cramps, and diarrhea.
Carcinoid tumour occur throughout the gastrointestinal tract, most commonly in the appendix, ileum and
rectum in decreasing order of frequency.
Right-sided valvular heart disease may develop after several years.
The syndrome results from vasoactive substances (including serotonin, bradykinin, histamine,
prostaglandins, polypeptide hormones) secreted by the tumor, which is typically a metastatic intestinal
carcinoid.
Diagnosis is clinical and by demonstrating increased urinary 5-hydroxyindoleacetic acid(HIAA).
Tumor localization may require a radionuclide scan or laparotomy.
Treatment of symptoms is with somatostatin or octreotide, but surgical removal is performed where
possible; chemotherapy may be used for malignant tumors.
Pathology
Question 40
Warthin finkeldey cells are seen in
A>Measles
B> Rubella
C> Rabies

D> Typhoid
Answer: Option A - Measles
Explanation: Multinucleated cells like Warthin Finkeldey are seen in Measles
Measles virus infects by invasion of respiratory epithelium.
Local multiplication leads to viremia (day 2-3), then spread to RE system.
Two types of Multinucleated giant cells in both epidermis & oral epithelium by 7-11 days.
Warthin Finkeldey cells of reticuloendothelial system
Epithelial giant cells of respiratory & other epithelia.

Warthin?Finkeldey cell:
Type of giant multinucleate cell found in hyperplastic lymph nodes early in the course of
measles
Under the light microscope, these cells consist of a large, grape-like cluster of nuclei.
Also with HIV-infected individuals and Kimura disease .
Rarely in neoplastic (e.g. lymphoma) & non-neoplastic lymph node disorders.
Unknown origin; Reports of staining with markers similar to follicular dendritic cells,
including CD21.

Question 41
CD59 marker of which disease

A> PNH
B> PTEN
C> BRR

D> Cowden syndrome
Answer: Option A - Paroxysmal nocturnal hemoglobinuria (PNH)
Explanation:
Paroxysmal nocturnal hemoglobinuria (PNH) is a disease, due to acquired mutations in
"Phosphatidylinositol GlycanComplementation Group A" gene (PIGA)..
Associated with deficiency of glycosylphosphatidylinositol (GPI) anchor proteins along with
absence of external surface membrane proteins attaching to it.
CD55 (DAF) and CD59 (MIRL) are two such complement defence proteins
CD59 deficiency:
Common finding in RBCs & WBCs of patients with chronic hemolysis suffering from PNH
Diagnosis:
The definite diagnosis based on demonstration of a substantial proportion of patient's RBC
having increased susceptibility to complement (C), due to the deficiency on their surface of
proteins (particularly CD59 & CD55)


Question 42
Opsonin is

A> C3a
B> C3b
C> C5a

D> C6
Answer: Option B - C3b
Explanation:
The process of coating a foreign particle targeting & preparing it for phagocytosis process is
"Opsonization". Substances involved are opsonins.

Main opsonins from complement system is C3

Examples of opsonins include:
? Antibodies:
o IgG and IgA
? Components of the complement system:
o C3b, C4b, and iC3b
? Mannose Binding Lectin (MBL):
o Initiates the formation of C3b
? Membrane Attack Complex (MAC)
o Includes C5b, C6, C7, C8 & polymeric C9

Opsonization & complement proteins:
Mainly C3b, iC3b & C4b
C3:
Most abundant protein of all complementary proteins,
Cleaves into C3a and C3b
C3a -
Binds and activates mast cells & basophils, release histamine.
C3b -
Most critical component in both classical & alternative pathway
C3b attaches to bacterial surfaces for opsonization by phagocytes

Question 43
Bernard?Soulier syndrome due to deficiency of
A> Gp 2b/3a
B> Gp 1b
C> vWf

D> TNF
Answer: Option B - Gp 1b
Explanation:
Bernard?Soulier syndrome (BSS) / Hemorrhagic Parous Thrombocytic Dystrophy
Rare autosomal recessive coagulopathy
Causes a deficiency of glycoprotein 1b (Gp1b), receptor for von Willebrand factor.

Question.44
Cowden syndrome

A> P53
B> PTEN
C> Rb


D> Ras
Answer: Option B - PTEN
Explanation:
"Phosphatase and Tensin" homolog (PTEN) - protein in humans encoded by the PTEN gene. Gene
mutations promotes development of cancers.
Cowden's disease / Multiple Hamartoma Syndrome -
Part of PTEN hamartoma tumor syndrome
An autosomal dominant syndrome
Trichilemmomas - Numerous tumors of hair follicles in face
Multiple hamartomatous polyps in GI tract.
Very high risk of breast & thyroid carcinoma
Treatment:
B/L mastectomies recommended
Contraindicated are mammography & other radiation exposure of breast tissue
Question. 45
Chromosome involved in myotonic dystrophy is

A>Chromosome 19
B> Chromosome 20
C> Chromosome 21
D> Chromosome 22
Answer: Option A - Chromosome 19
Explanation:
Myotonic dystrophy is transmitted by mutation in an 'unstable trinucleotide repeat sequence' in
gene 19q 133.
Features:
An autosomal dominant disorder
Most common adult muscular dystrophy
Characteristics feature:
Myopathy is distal (in contrast to other myopathies - mostly proximal).
Muscle atrophy selectively involves type I fibres only
Appears by 5 years, causes a slow relaxation of hand grip following a forced voluntary
closure.

Question. 46
TRALI occurs within how many hours of transfusion?

A> 48 Hrs
B> 72 Hrs

C>6 Hrs
D> 12 Hrs
Answer: Option C - 6 Hrs
Explanation:
Transfusion-Related Acute Lung Injury (TRALI) - Syndrome characterized by acute respiratory
distress following transfusion.
Symptoms:
Typically develop during, or within 6 hours of transfusion.
Rapid onset of dyspnea & tachypnea.
Associated fever, cyanosis, & hypotension.
Clinical examination:
Reveals respiratory distress.
Pulmo-nary crackles may be present with no signs of CHF or volume overload.
CXR - Evidence of B/L pulmonary edema unrelated to CHF (non-cardiogenic pulmonary
edema),
Bilat-eral patchy infiltrates rapidly progressing to complete "white out"
indistinguishable from Acute Respi-ratory Distress Syndrome (ARDS).

Question. 47.
Kidney responds to shock by

A> Decreases renal blood flow
B> Increases afferent arteriole resistance
C> GFR remains unaltered
D> Perfusion of kidney increases
Answer: Option B - Increases afferent arteriole resistance
Explanation:
Kidney utilizes the following mechanisms as a response to shock:
Release of aldosterone from hypoxic kidney
Release of ADH due to decreased effective circulating blood volume.
Reduced GFR due to arteriole constriction
Tissue fluid shift into plasma due to lowered hydrostatic pressure (Hypotension)

Question. 48
Choose the best method of diagnosis for the clinical sign represented in the image.





A> Serum copper
B> Serum ceruloplasmin
C> Karyotyping

D> PCR
Answer: Option B - Serum Ceruloplasmin
Explanation:
Image represents "Kayser-Fleischer" ring
KF ring is golden brown ring due to deposition of copper in Descement's membrane of cornea.
Clinical feature of "Wilson's disease"
Wilson's Disease:
Rare autosomal recessive disease.
Characterized by abnormal copper metabolism
Ophthalmic presentation:
Deposition of copper in posterior capsule of lens results in sunflower cataract.
Diagnosis:
Sternlieb's criteria:
Presence of KF rings
Decreased serum ceruloplasmin (copper containing enzyme / ferroxidase) levels
Measuring hepatic copper levels in liver biopsy
24-Hr urine copper excretion
Kayser Fleischer ring:
Excess circulating copper deposits in descemet's membrane.
Usually golden brown located in peripheral cornea, beginning at schwalbe's line upto 5
mm into cornea.
Detected via Gonioscopy in earlier stage & seen by naked eyes in advanced stage.

Question. 49
Which of the following is epithelial tumor of stomach?

A> Carcinoid
B> Lymphoma
C> GIST

D> Gastric adenocarcinoma
Answer: Option D - Gastric adenocarcinoma
Explanation:
Malignant epithelial tumor originating from glandular epithelium of gastric mucosa.

Aggressively invade the gastric wall.
Lauren classification:
Two types of gastric adenocarcinoma are present.
Intestinal type
Diffuse type
Intestinal type - Irregular tubular structures
Diffuse type - Mucinous & colloidal "Leather-bottle stomach"

Question. 50
Identify an X linked disorder?

A> Color blindness
B> Thalassemia
C>Azoospermia

D>Retinitis Pigmentosa
Answer: Option A - Color blindness
Explanation:
Hereditary color blindness/ Achromatopsia
The ability to appreciate one or more primary color is defective (anomalous) or absent (anopia)
Due to mutations in X chromosome
Red & green pigment cones coded by X chromosome; Blue coded on chromosome 7
More common in males than females
Acquired - (Optic nerve/ macular damage)
Ishihara chart -
Test red/ green color blindness - Farnsworth 100 hue test
Other options:
Azoospermia&Retinitis Pigmentosa - Y-chromosome linked disorder
Thalassemia - Inherited (Autosomal recessive pattern) blood disorders characterized by abnormal
hemoglobin production. Genes in Chromosome 11 and 16 involved.

Question. 51.
H And L variety seen in

A> Mixed cellularity hodgkin
B> Lymphocyte depleted
C> Lymphocyte predominance

D> Nodular sclerosis
Answer: Option C - Lymphocyte predominance
Explanation:
Hodgkin lymphoma (HL) - common "Malignant Lymphomas"
2 entities:
Classical HL (cHL)
Nodular Lymphocyte?predominant HL (NLPHL).

Cells :
Classical HL - Hodgkin and Reed/Sternberg (HRS) cells
Nodular Lymphocyte predominant HL - Lymphocytic & Histiocytic (L&H) cells

Question. 52
Stellate granuloma seen in

A> Sarcoidosis
B> Cat scratch disease
C> Cryptococcosis

D> Histoplasmosis
Answer: Option B - Cat scratch disease
Explanation:
Bacterial infection causes by Bartonella henselae
Acquired infected cat/kitten scratch
Histology:
Characterized by granulomatous inflammation of lymph nodes.
Skin lesion demonstrates a circumscribed focus of necrosis
Regional lymph nodes demonstrate follicular hyperplasia with central stellate necrosis with
neutrophils, surrounded by palisading histiocytes (suppurative granulomas) & sinuses
packed with monocytoid B cells, usually without perifollicular and intrafollicular epithelioid
cells.

Question. 53
Which option best explains "Flipping effect"?

A> LDH 1 > LDH 2
B> LDH2 > LDH1
C> LDH 2 > LDH 3

D> LDH 3 > LDH 2
Answer: Option A - LDH 1 > LDH 2
Explanation:
Lactate dehydrogenase, tetrameric enzyme with 4 subunits,
4 Subunits with 2 isoforms - H isoform (Heart) & M isoform (Muscle)
Heart & RBCs - LDH-1 (4H);
Reticuloendothelial system - LDH-2 (3H1M)
Lungs - LDH-3 (2H2M)
Kidneys, placenta, & pancreas - LDH-4 (1H3M)
Liver & striated muscle - LDH-5 (4M)
Uses:
LDH levels are more in RBC
Helpful in assessment of Hemolysis / Tissue breakdown
Flipping effect:




Usually LDH 2 in predominant in serum & LDH 1 is predominant in heart
Higher levels of LDH 1 than LDH 2 (Flipped pattern) is suggestive of myocardial infarction
Damaged cardiac tissues release LDH 1 into bloodstream.

Question. 54
Nude mice is not resistant to xenograft due to absence of

A> B cell
B> T cell
C> Both b and t cell
D> None
Answer: Option B - T cell
Explanation:
Nude mice lacks "Thymus" & cannot generate mature T lymphocytes.
Absence of T-lymphocytes makes it unable to mount adaptive immune responsesrequiring CD4,
helper T cells, CD8 and cytotoxic T cells.
Adaptive immune responses that remain unresponsive to nude mice include:
Antibody formation (CD4 + helper T cells)
Cell-mediated immune responses (CD4+ and/or CD8+ T cells)
Delayed-type hypersensitivity responses (CD4+ T cells)
Killing of virus-infected or malignant cells (CD8 + cytotoxic T cells)
Graft rejection (both CD4+ & CD8+ T cells)
Uses:
Laboratory study animal - Insights into immune system, leukemia, solid tumors, AIDS & other
immune deficiency diseases.
Absence of functioning T cells prevents them rejecting the allografts & Xenografts.

Question. 55
Identify the condition represented in the image.


A> Seminoma
B> Germ cell differentiate tumor
C> Non-seminoma

D>Teratoma
Answer: Option A - Seminoma

Explanation:
Seminoma - Germ cell tumor of testicle/ rarely mediastinum or other extra-gonadal locations.
Malignant neoplasm;
One of most treatable & curable cancers;
Survival rate >95% if discovered in early stages.
Usually unaffected fertility & other sexual functions remains intact.
Originates in germinal epithelium of seminiferous tubules
Histology:
Typically prominent lymphocytic infiltrate in the fibrous stroma separating the clusters of
tumor cells.
Treatment:
Requires removal of one testicle.

Question. 56
Anaplasia is

A> Changing one type of epithelium to another
B> Nuclear chromatin
C> Lack of differentiation
D> Morphological changes
Answer: Option C - Lack of differentiation
Explanation:
Refers to a lack of differentiation in neoplastic cells.
Well-differentiated tumors resemble their tissue of origin
Poorly-differentiated or undifferentiated (anaplastic) tumor cells appear primitive and lack
specialization along any particular cell line.
Question. 57
Which level of prolactin definitely suggest prolactinoma

A> 200 ng/ml
B> 150 ng/ml
C> 50 ng/ml
D> 100 ng/ml
Answer: Option C - 200 ng/ml
Explanation:
Prolactinomas /"Prolactin-producing adenomas" /"Lactotroph adenomas"
Benign tumors of pituitary gland producing prolactin
Higher blood prolactin concentrations
The diagnosis of a prolactinoma is confirmed by demonstrating persistently elevated blood levels
of prolactin over 150-200 ng/ml





Question. 58
Laxative abuse causes which of the following renal stones ?

A> Uric acid
B> Ammonium urate
C> Struvite
D> Ca oxalate
Answer: Option B - Ammonium urate
Explanation:
Based on the chemical nature two types of kidney stones:
Calcium oxalate (majority).
Others include Uric acid, Struvite (Infected stones), and Cystine stones (rare hereditary
metabolic disorder
Characteristic stone formation in laxative abuse:
Laxative abuse acts a factor in kidney stone formation.
Laxative abuse causes potassium loss
As a compensation mechanism kidney produces large amount of ammonium.
Resulting in formation of uncommon stone type - ammonium acid urate.

Question. 59
Identify the condition shown in the image.


A> Lobar Pneumonia
B> Bronchopneumonia
C> Acute glomerulonephritis
D> Congested kidney

Answer: Option A- Lobar Pneumonia
Explanation:
Image depicts the red hepatization of lungs
Red hepatization is seen in conditions likelobar pneumonia.
Lobar Pneumonia:.
Pneumococcal pneumonia - Most common cause of lobar pneumonia
Progresses from a red hepatization phase to a gray hepatization phase
Red hepatization:



Characterized by consolidation of airspaces of lungs.
C/S reveals lungs appear brown-red, firm, & airless.resembling liver.
Histological:
Congested alveolar capillaries & alveolar spaces are filled with erythrocytes,
neutrophils, and fibrin.
Red cells disintegrate, with persistence of the neutrophils and fibrin.
Question. 60
Identify the condition represented in below image


A> Miliary tuberculosis
B> Bronchiectasis
C> COPD
D> Lung cancer

Answer: Option A- Miliary tb
Explanation:
Miliary tuberculosis (TB) is widespread dissemination of Mycobacterium tuberculosis
Result of hematogenous spread.
Distinctive pattern seen on a chest radiograph
Tiny sized lesions (1?5 mm), seen as tiny spots distributed throughout lung fields
appearing similar to millet seeds
Hence, the term "miliary" tuberculosis.
Miliary TB may also infect liver & spleen.

Question. 61
Which of the following can result in dactylitis

A> Hemophilia
B> Von willebrand disease 1
C> Measles

D> Sickle Cell Anemia

Answer: Option D - Sickle Cell Anemia
Explanation:
Dactylitis (Hand-Foot Syndrome) is seen in sickle cell anemia

Severe pain affecting the bones of hands, feet, or both.
Often 1st symptom of sickle cell anemia in babies.

Question. 62
Which chromosome is responsible for the production of MIF?
A> Chromosome 16
B> Chromosome 22
C> X Chromosome

D> Y chromosome

Answer: Option B - Y Chromosome
Explanation:
Anti-Mullerian Hormone (AMH) / Mullerian Inhibiting Factor (MIF); Mullerian-inhibiting Hormone
(MIH) / Mullerian-inhibiting Substance (MIS).
AMH - Downstream genes regulated by SRY pathway
SRY - Gene in "Sex determining region" - short arm of Y chromosome - Testis determining factor.
Secreted by Sertoli cells of the testes.
The production of AMH is controlled by two autosomal gene loci.
Hormone code
Receptor code.
Glycoprotein hormone
Related to inhibin & activin
Member of the transforming growth factor- (TGF-)
Key roles are in growth differentiation and folliculogenesis.
Microbiology
Question 63
Burkholderia cepacia is resistant to which of the following drugs:

A>Ceftazidime
B>Trimethoprim-sulfamethoxazole
C>Temocillin
D>Cefotetan
Answer: D
Explanation: B. cepacia complex strains are intrinsically resistant to a wide range of antimicrobial agents,
including aminoglycosides, polymyxin, first and second generation cephalosporins, and carboxypenicillins
Antimicrobial agents that are effective against B. cepacia complex include meropenem, ceftazidime,
piperacillin, temocillin, and trimethoprim-sulfamethoxazole.


Question 64
Shingles Is caused by which of the following ?

A. Varicella-zoster
B. Herpes simplex
C. CMV
D. None


Ans. A. Varicella-zoster
Explanation:
Shingles, also called herpes zoster, is a painful skin rash.
Shingles is caused by reactivation of the varicella zoster virus, the same virus that causes
chickenpox.
Question 65
Urea breath test is used for diagnosis of:

A>H.pylori
B>Campylobacter jejuni
C>E. coli
D>Lactobacillus
Answer: A
Explanation: The urea breath test is a rapid diagnostic procedure used to identify infections by
Helicobacter pylori, a spiral bacterium implicated in gastritis, gastric ulcer, and peptic ulcer disease. It is
based upon the ability of H. pylori to convert urea to ammonia and carbon dioxide
Question 66
Hyperacute graft rejection occurs after how much time?

A>24 hours
B>2 weeks right
C>In minutes
D>Years
Answer: C
Explanation : Hyperacute Transplant Rejection occurs almost immediately and is often evident while you
are still in surgery. It is caused by accidental ABO Blood type mismatching of the donor and recipient which
almost never happens anymore..Acute onset is in few weeks to month. Chronic onset is from months to
years.


Question 67
Australian antigen for hepatitis b is?

A>Hb S ag
B>Hb E ag
C>Hb D ag
D>HbV Dna
Answer: A
Explanation: HBsAg (also known as the Australia antigen) is the surface antigen of the hepatitis B virus
(HBV). It indicates current hepatitis B infection.
Question 68
Which fungus is most commonly associated with orbital cellulitis in patients with diabetic ketoacidosis.

A>Candida
B>Mucor
C>Aspergillus
D>Rhizopus
Answer: C
Explanation: Orbital cellulitis term is reserved for infections behind the orbital septum which may or may
not spill over to lids. Bacterial OC is more common in children and fulminant infection (& ischemic
infarction) with Mucor or Aspergillus typically affectspatients with diabetes (esp ketoacidosis) and
immunosuppression. Presentation is Extensive swelling of lids with chemosis often obscure proptosis (i.e.
most commonly lateral & downwards). Proptosis with impaired mobility resulting in diplopia Pain is severe,
increased by movement of eye or pressure Unilateral, tender, warm & red periorbital edema,painful
ophthalmoplegia
Question 69
Sabin Feldman dye test is used for diagnosis of which of the following condition:

A>Botulism
B>Toxoplasmosis
C>Sarcoidosis
D>Yellow fever
Answer: B
Explanation: A Sabin?Feldman dye test is a serologic test to diagnose for toxoplasmosis
Question 70
Acute Hemorrhagic Conjunctivitis is caused by which of enterovirus type ?


A>69
B>68
C>70
D>71

Answer: C
Explanation: Acute hemorrhagic conjunctivitis (AHC) is characterized by conjunctival congestion, vascular
dilatation, and onset of edema .Serologic studies have been useful in showing the presence of neutralizing
antibodies to Coxsackie group A24 (CA24) and enterovirus E70 (EV70) strains as the causative agent.
Question 71
Echinococcus granulosus are commonly seen in which of the given animals:

A>Dog
B>Cat
C>Fox
D>Pig

Answer: A
Explanation: Echinococcus granulosus, also called the hydatid worm, hyper tape-worm or dog tapeworm.
Domestic dogs (Canis familiaris) have been recognised as the definitive host of the parasite.
Question 72
An anaerobe causing multiple abscess with discharging sinuses, demonstrating sulphur granules in pus
is?

A>Actinomycetes
B>Nocardia
C>Salmonella
D>Tularemia
Answer: A
Explanation: Multiple abscess with discharging sinuses, demonstrating sulphur granules in pus are
characteristics of actinomycetes
Question 73
Whole blood is used as a sample for which test?

A>Bacteria
B>IGRA
C>Genexpert
D>Virus
Answer: B

Explanation: Interferon-Gamma Release Assays (IGRAs) are whole-blood tests that can aid in diagnosing
Mycobacterium tuberculosis infection
Question 74
Which organism causing acute bacterial prostatitis ?

A>Enterococcus
B>Streptococcus viridans
C>Peptostreptococcus
D>E.coli
Answer: D
Explanation: Aerobic gram-negative bacilli are the predominant pathogens in bacterial prostatitis. E. coli
cause 50%?80% of cases; other pathogens include Enterobacteriaceae (eg, Klebsiella and Proteus, which
account for 10%?30% of cases), Enterococcus species (5%?10%)
Question 75
Which of the following organism releases histamine and cause scombroid fish poisoning

A>Salmonella
B>Staphylococcus
C>P. aeruginosa
D>Weissella
Answer: C
Explanation: Scombroid poisoning is one of the most common causes of morbidity associated with fish
intake which have not been refrigerated properly from the time they were caught until the time they were
served. Bacteria act on compounds in the fish, releasing histamine. Process is induced by enzymes
produced by primarily enteric gram-negative bacteria (e.g., Morganella morganii, Escherichia coli,
Klebsiella species and Pseudomonas aeruginosa) found in the fish's cutis and intestines.,.
Question 76
Who is the father of microbiology?

A>A.V.L.hook
B>Robert brown
C>J.C Bose
D>Pasteur
Answer: A
Explanation: "Antoni van Leeuwenhoek" is commonly known as "the Father of Microbiology.
Question 77
Cutaneous larva migrans caused by which organism?


A>Strongyloides
B>Toxocara canis
C>Ancylostoma braziliense
D>Necator americanus
Answer: C
Explanation: Cutaneous larva migrans(CLM)
/It is a skin disease in humans, caused by the larvae of various nematode parasites of the
hookworm family (Ancylostomatidae).
The most common species causing this disease in the Americas is Ancylostoma braziliense.
These parasites live in the intestines of dogs, cats and wild animals and should not be confused with
other members of the hookworm family for which humans are definitive hosts, namely
Ancylostoma duodenale and Necator americanus.
Colloquially called creeping eruption due to its presentation, the disease is also somewhat
ambiguously known as "ground itch" or (in some parts of the Southern USA) "sandworms", as the
larvae like to live in sandy soil.
Another vernacular name is plumber's itch.
The medical term CLM literally means "wandering larvae in the skin"
Question 78
After kidney transplantation which organisms infection is more likely to happens

A>CMV
B>Klebsiella
C>Streptococcus
D>Staphylococcus
Answer: A
Explanation: CMV is the most common viral infection after Kidney Transplantation. Most common CMV
syndrome in kidney transplant patient is fever (most common), leukopenia, hepatosplenomegaly, myalgia
and arthralgia.
Question 79
Microbial cause of catheter infection in young female.

A>S.aureus
B>S.viridans
C>P. mallei
D>P. cepacia

Answer:
Explanation: Etiology of catheter related infection




Microorganism
Percentage
Coagulase negative staphylococci
30 - 40
Staph aureus
5 - 10
Enterococci
4 - 6
Candida spp.
3 - 6
Pseudomonas aeruginosa
2 - 5
Enterobacter spp
1 - 4
Acinetobacter spp.
1 - 2
Serratia spp.
< 1
Others
< 1 - 5
Question 80
Identify the organism related to blood smear image


A>F.malariae
B>S. Typhi
C>Treponema pallidum
D>Toxoplasma gondii.
Answer: A
Explanation: Malaria parasite can be seen in blood smear of patients affected by the disease at specific
stages in the disease. The thick smears allow detection whereas the thin smears allow identification of the
specie. This thin smear shows multiple malaria trophozoites inside red blood cells (Early ring form
). However, trophozoites can also be seen outside red blood cells.



MEDICINE
Question. 81
Which of the following is endogenous pyrogen

A> PG E2
B> PG D2
C> PGF2 alpha

D> PG I2
Answer: Option -A
Explanation: Pyrogene
Pyrogene are substances that cause fever.
Pyrogens may be exogenous or endogenous
Exogenous Bacterial toxins
Endogenous IL-1, TNF-a, IL-6, Interferons, Ciliary neurotrophic factor
These pyrogene increase the level of PGE, in the hypothalamus that elevates the thermoregulatory set
point and causes fever.
Question. 82.
In Bartter syndrome defect is seen in:

A> Defect in PCT
B> Defect in DCT
C> Defect in thick ascending limb of loop of henle

D>None
Answer: Option C - Defect in thick ascending limb of loop of henle
Explanation:
Autosomal recessive disorder.
Genetic defect in the thick ascending limb of the loop of henle
Defects in Na-K-2C/ coLtransporter, K or Cl
channels result in lack of concentrating ability
Question. 83.
Not seen in allergic pulmonary aspergillosis is -

A> High IgE level
B> Recurrent pneumonia
C>Occurrence in patients with old cavitary lesions
D>Pleural effusion
Answer: Option B- Recurrent pneumonia
Explanation:

Allergic bronchopulmonary aspergillosis is a pulmonary hypersensitivity disorder caused by allergy to
fungal antigens that colonize the tracheobronchial tree.
It most commonly occurs in atopic asthmatic individuals in response to antigen of aspergillus species.
Main diagnostic criteria
Clinical history of Asthma Q
Pulmonary infiltrates (transient /fleeting or fixed)Q
Peripheral eosinophilia (> 1000 /,uL)Q
Immediate skin reactivity to Aspergillus antigen (wheal and flare response)
Serum precipitins to A. fumigatus
Elevated serum IgE levels(>100Ong/ml)
Central /proximal bronchiectasis
Secondary diagnostic criteria
History of brownish plugs in sputum
Identification / culture of A., fumigatus from sputum
Late skin reactivity to aspergillus antigen - CMDT
Elevated IgE (and IgG) class antibodies specific for A. fumigatus - Harrisons
Note
Elevated IgE (and IgG) class antibodies specific for A. fumigatus has been mentioned as a secondary
diagnostic criteria in Harrison's textbook while Fishman's textbook includes this as a main/major diagnostic
criteria.
Question. 84
Pseudo P Pulmonale

A> Hypokalemia
B> Hyponatremia
C> Hypocalcemia

D> Hypercalcemia
Answer: Option -A
Explanation: In some cases there can be a notched (or bifid) p-wave known as "p mitrale", indicative of left
atrial hypertrophy which may be caused by mitral stenosis. There may be tall peaked p-waves. This is called
"p-pulmonale" and is indicative of right atrial hypertrophy often secondary to tricuspid stenosis or
pulmonary hypertension.
A similar picture can be seen in hypokalemia (known as "pseudo p-pulmonale").
Question. 85
Automatic Implantable Cardioverter Defibrillator, (AICD) implantation is done for which of following
conditions:


A> Brugada syndrome
B> Ventricular fibrillation
C> Acute coronary syndrome with low EF
D> ALL
Answer: Option D all of the above
Explanation:
An Automatic Implantable Cardioverter Defibrillator, (AICD), is a small electronic device that is implanted
into your chest to monitor and correct an abnormal heart rhythm, or arrhythmia. These devices are used to
treat serious and life-threatening arrhythmias and are the most effective way of doing so.
Brugadasyndrome is a condition that causes a disruption of the heart's normal rhythm.
Ventricular fibrillation is a heart rhythm problem that occurs when the heart beats with rapid, erratic
electrical impulse.
Acute coronary syndrome is a term used to describe a range of conditions associated with sudden,
reduced blood flow to the heart.

Question . 86
What is the line of treatment for intractable Sydenham chorea?

A> Haloperidol
B> Valproate
C> Warfarin

D> Risperidone
Answer: Option B - Valproate
Explanation:
Sydenham's chorea (SC) / Chorea minor / Rheumatic chorea (RC) / St. Vitus' s Dance
Major diagnostic criteria for rheumatic disease
Most common acquired chorea during childhood
Characterized by rapid, uncoordinated jerking movements primarily affecting the face,
hands and feet
Benzodiazepines facilitate the action of GABA and valproate enhances the action on GABA, hence
these agents are used to treat chorea
Question. 87
Neurofibromatosis 1 criteria except

A> Brain tumor
B>Acoustic neuromas.
C> Pseudoarthrosis
D> Cafe-au-lait spots
Answer: Option B - Acoustic neuromas
Explanation:

Clinical diagnosis requires presence of at least 2 of 7 criteria to confirm presence of
neurofibromatosis, type 1.
Despite suspicion, most signs do not appear until late childhood / adolescent
The 7 clinical criteria used to diagnose NF1 are as follows:
Six or more caf?-au-lait spots / hyperpigmented macules greater than or equal to 5 mm in
diameter in prepubertal children & 15 mm postpubertal
Axillary or inguinal freckles (>2)
Two or more typical neurofibromas or one plexiform neurofibroma
Optic nerve glioma
Two or more iris hamartomas (Lisch nodules) (often identified only through slit-lamp
examination)
Sphenoid dysplasia or typical long-bone abnormalities such as pseudarthrosis
Strong family history (1st degree relative with NF1)
Question. 88
Most common site of chronic gastric ulcer

A> Pyloric antrum
B> Upper part of lesser curvature
C> Lower part of lesser curvature
D> Segment of large intestine
Answer: Option A - Pyloric antrum
Explanation:
A major causative factor (60% of gastric& up to 50?75% of duodenal ulcers) is chronic inflammation
due to Helicobacter pylori that colonizes the antral mucosa.
Question. 89
Approximate time interval between HIV infection & manifestation of AIDS is?

A> 7.5 yr
B> 10 yr
C> 12 yr

D> 11 yr
Answer: Option B - 10 years
Explanation: The approximate time frame required for incubation is usually 10 years.
Question. 90
Heller's myotomy is done for

A> Zenker's diverticulum
B> Achalasia cardia

C> Bunions

D> Knee arthroscopy
Answer: Option B - Achalasia cardia
Explanation:
Achalasia, a disorder of esophagus characterized by progressive inability to swallow solids & liquids.
Causes include weakened esophageal muscles & issues with lower esophageal sphincter relaxation
Heller Myotomy, surgical procedure offers long term symptomatic relief to these patients.
It involves weakening of muscles at gastroesophageal junction, allowing the valve between
oesophagus & stomach to remain open.
Question. 91
Myocardial stunning pattern not matching the ECG. What is the diagnosis.
A> Takotsubo cardiomyopathy
B> Restrictive cardiomyopathy
C> Brigade's cardiomyopathy

D> Pericardial something
Answer: Option A -Takotsubo cardiomyopathy
Explanation:
"Myocardial Stunning" is a state where certain segments of myocardium (corresponding to area of
major coronary occlusion) shows forms of contractile abnormality.
A segmental dysfunction persisting for a variable period of time, about two weeks, even after
ischemia has been relieved (by for instance angioplasty or coronary artery bypass surgery).
Takotsubo cardiomyopathy / Takotsubo syndrome, a temporary condition where your heart muscle
becomes suddenly weakened or 'stunned'.
Question.92
Alternative drug for cardiac arrest in place of epinephrine is?

A> Amiodarone infusion
B> Atropine
C> High dose vasopressin
D> Adenosine

Answer: Option C - High dose vasopressin
Explanation:
Vasopressin is an alternative vasopressor at high doses, causes vasoconstriction by directly
stimulating smooth muscle V1 receptors.

American Heart Association (AHA) guidelines states "Vasopressin is a reasonable first-line
vasopressor in patients with ventricular fibrillation or pulseless ventricular tachycardia.
Additionally, the guidelines comment that one dose of vasopressin 40 U may replace the first or
second dose of epinephrine in all pulseless sudden cardiac arrest scenarios, including asystole and
pulseless electrical activity.


Question. 93
Patient presenting with cutaneous vasculitis, glomerulonephritis, peripheral neuropathy, Which
investigation is to be performed next that will help you diagnose the condition?

A> ANCA
B> RA factor
C> Hbsag

D> MIF
Answer: Option A - ANCA
Explanation:
Anti-neutrophil cytoplasmic antibodies (ANCAs):
Group of autoantibodies
IgG type mainly,
Produced against antigens in cytoplasm of neutrophil granulocytes & monocytes.
Particularly associated with systemic vasculitis, so called "ANCA-associated vasculitis".
Question. 94
Cryoglobulinemia

A> Hepatitis c
B> Ovarian cancer
C> Diabetes
D> Leukemia
Answer: Option A - Hepatitis C
Explanation:
Cryoglobulinemia / Cryoglobulinemic disease with large amounts of cryoglobulins in blood
Cryoglobulins are proteins (mostly immunoglobulins themselves)
Become insoluble at reduced temperatures.
Mixtures of monoclonal or polyclonal IgM, IgG, and/or IgA & blood complement proteins like C4
are associated with cases of infectious diseases, particularly Hepatitis C infection,
Question.95
Causes of hypokalemic metabolic alkalosis with hypertension

A> Liddle syndrome
B> Bartter syndrome
C> Gitelman syndrome

D> Renal tubular acidosis
Answer: Option A - Liddle syndrome

Explanation:
Liddle syndrome - Rare hereditary disorder
Increased activity of the epithelial sodium channel (E-Na Ch)
Activating kidneys to excrete potassium retaining excessive sodium & water, leading to
hypertension.
Question. 96
Gold criteria for very severe COPD

A> Fev1/Fvc <70 and Fev1 < 30
B> Fev1/Fvc <70 and Fev1 < 70
C> Fev1/Fvc <70 and Fev1 < 50

D> Both A and C
Answer: Option A - Fev1/Fvc <70 and Fev1 < 30
Explanation:
COPD should be considered in any patient who has dyspnea, chronic cough or sputum production,
and/or a history of exposure to risk factors for the disease.
Spirometry is required to make the diagnosis.
Presence of post-bronchodilator FEV1/FVC < 0.70 confirms the presence of persistent airflow
limitation.
Stage IV / Very Severe COPD
o Severe airflow limitation (FEV1/FVC < 70%; FEV1 <. 30% predicted) or FEV1 < 50% predicted
plus chronic respiratory failure.
o Patients may have Very Severe (Stage IV)COPD even if FEV1 is > 30% predicted.
Question. 97
ABPI increases artificially in

A> Arteriosclerosis calcified arteries
B>Ischemic ulcers
C> Intermittent claudication
D> DVT
Answer: Option A - Arteriosclerosis calcified arteries
Explanation:
The ankle-brachial pressure index (ABPI) / Ankle-Brachial index (ABI):
Ratio of blood pressure at ankle to blood pressure in upper arm (brachium).
Compared to arm, lower blood pressure in leg suggests blocked arteries due to peripheral artery
disease (PAD).
Ankle brachial pressure index (ABPI) is a method for the quantification of peripheral vascular
disease that results from advanced atherosclerosis.



Question. 98
Minimal dysfunction syndrome seen in

A> Dyslexia
B> ADHD
C> Mental subnormality

D> Down's syndrome
Answer: Option A - Dyslexia
Explanation: Clinical feature of Minimal dysfunction syndrome includes dyslexia.
Minimal brain dysfunction:
o Neurodevelopmental disorder.
o Characterized by evidences of immaturity involving control of activity, emotions, & behavior
o Specific learning disabilities involving the communicating skills needed in reading, writing, and
mathematics.
o Inability to maintain attention & concentration
o Inability to skillfully blend the auditory & visual functions essential in language performance

Pharmacology
Question.99
Site of action of amphotericin B is:

A> Ribosomes
B> Cell wall
C> Plasma membrane
D> Protein

Answer: Option - B - Cell wall
Explanation: Polyene drug compounds like Amphotericin B acts on cell membrane
Amphotericin B, antifungal agent.
Chemically a polyene compound
Obtained from Streptomyces nodosus.
Polyenes have a high affinity for "ergosterol" present in the fungal cell membrane.
Binds and gets inserted into the cell membrane forming "Micropore".
Marked increase in permeability of cell membrane.

Question. 100
Which antiretroviral drug also has anti hepatitis activity?

A> Abacavir
B> Tenofovir
C> Nevirapine

D> Emtricitabine

Answer: Option D - Emtricitabine
Explanation: Emtricitabine, NRTI drug with both antiretroviral & anti-hepatitis properties
Nucleoside reverse transcriptase inhibitor for the prevention and treatment of HIV infection in
adults and children.
Also used in combination with tenofovir

Question. 101
Drug of choice for resistant rheumatic chorea?

A> Valproate
B> Haloperidol
C> Diazepam
D> Probenecid

Answer: Option - A - Valproate
Explanation:
Valproate, Sulpiride, & diazepam are used for symptomatic treatment.
Acute Rheumatic Fever:
Abnormal immune response to group A streptococcal infection
Commonly affecting the joints, heart, brain, and skin.
Symptoms: Arthritis, related to carditis &chorea.
Sydenham's chorea / Chorea minor
Characterized by rapid, uncoordinated jerking movements primarily affecting the face,
hands and feet.
Signs & symptoms of chorea usually do not respond well to treatment with antirheumatic
agents
Symptomatic treatment include anticonvulsants (eg, valproate, carbamazepine) and
neuroleptics (eg, pimozide, haloperidol, risperidone, olanzapine

Question. 102
At pKa=pH

A> Conc. of drug is 50% ionic and 50 % non-ionic
B> Absorption of drug is 50% ionic and 50% ionic
C> Conc of drug is 75% ionic and 25 % non-ionic
D> Conc of drug is 25% ionic and 75 % non-ionic

Answer: Option A - Concentration of drug is 50% ionic & 50 % Non-ionic
Explanation:
Numerically equal pKa & pH represents 50% drug ionization
pKa is negative logarithm of acidic dissociation of weak electrolyte.
On equal concentrations of ionized & unionized drugs, log 1 is zero.
Thus, when pKa is numerically equal to pH

(pKa=pH) ------------> 50 % drug is ionized.

Question. 103
Physiological dose of hydrocortisone (mg/kg/day) is

A> 5 mg/kg/day
B> 10 mg/kg/day
C> 15 mg/kg/day
D> 20 mg/kg/day

Answer: Option - B - 10 mg/kg/day
Explanation:
The normal rate of secretion of two principle corticoids
Hydrocortisone - 10 mg/kg/day (nearly half in morning hours)
Aldosterone - 0.125 mg/daily

Question 104
What is mechanism of action of colchicine in acute gout?

A> Inhibition of purine metabolism
B> Inhibition of uric acid conversion
C> Migration of leukocytes
D> Leukocytes, lymphocytes inhibition & microtubular inhibitor.
Answer: Option D - Leukocytes, Lymphocytes migration inhibition & microtubular inhibitor.
Explanation:
Colchicine acts by inhibiting the granulocyte migration into the inflamed joint.
An alkaloid from Colchium autumnale
Specifically suppresses gouty inflammation.
Doesn't inhibit the synthesis or promote the excretion of uric acid.
Mechanism of action:
Colchicine acts by,
inhibits the release of glycoprotein
Binds to fibrillar protein tubules inhibiting granulocyte migration into the inflamed joint.
An acute attack of gout starts by precipitation of urate crystals in synovial fluid.
Inflammatory response starts with granulocyte migration into joint
Phagocytosing urate crystals releasing glycoprotein
Glycoprotein increases lactic acid production and releasing lysosomal enzymes causing more joint
destruction.

Question. 105
Basiliximab is an

A> IL-1 receptor antagonist

B> Anti-CD3 antibody
C> IL-2 receptor antagonist
D> TNF inhibitor

Answer: Option - C - IL-2 receptor antagonist
Explanation:
Basiliximab exhibits high affinity towards IL-2 receptor, inhibiting it.
Anti- CD-25 antibody
High affinity for IL-2 receptor
Short plasma half life - 1 week
Useful in preventing transplant rejection reactions.
Adverse effects - Anaphylactic reactions & opportunistic infections.
Question.106
Pirenzapine is used

A> Gastric ulcer
B> Glaucoma
C> Hypertension
D> Congestive cardiac failure

Answer: Option A - Gastric Ulcer
Explanation:
Pirenzepine, a selective M1 anticholinergic drug inhibiting gastric acid secretion.
Low therapeutic dose range.
Used for treating gastric ulcer.

Question. 107
Which of the following antipsychotic have increased prolactin secretion

A> Olanzapine
B> Ziprasidone
C> Clozapine
D> Risperidone

Answer: Option - D - Risperidone
Explanation:
Significant rise in prolactin levels during risperidone therapy is observed.
Risperidone - Antipsychotic drug with combined 5-HT 2a and dopamine D2 antagonist activity
High affinity to alpha1, alpha 2 and H1 receptors
More potent D2 blocker than clozapine
Ameliorates symptoms of schizophrenia
Prolactin levels rise during risperidone therapy, but less epileptogenic than clozapine.
Produces extrapyramidal side effects are less only at lower doses (<6 mg/day).
Blockade of these contribute to efficacy and side effects like postural hypotension.

Frequently causes agitation.

Question. 108
Which of the following is glucocorticoid synthesis inhibitor?
A> Mifepristone
B> Flutamide
C> Finasteride
D> Metyrapone

Answer: Option D - Metyrapone
Explanation:
Inhibits 1113- Hydroxylase in adrenal cortex
Prevents the synthesis of hydrocortisone


Question. 109
Which of the following statements is incorrect w.r.t Prasugrel?

A> Not a prodrug
B> P2Y purinergic receptor blocker
C> Has a strong antiplatelet activity
D> Causes intracranial hemorrhage in TIA patients.

Answer: Option A - Not a prodrug
Explanation:
Prasugrel is a prodrug , similar to Clopidogrel
Thienopyridine drug class
Irreversible antagonist of P2Y12 ADP receptors
Rapidly absorbed, completely activated & exerts more consistent platelet inhibition.
Strong anti-platelet activity
Bleeding complications are more serious and frequent.
Contraindicated in patients with history of ischemic strokes and TIA's

Question. 110
Q-T elongation is seen in which drug?

A> Quinidine
B>Amiodarone
C>Magnesium Sulfate
D>Lignocaine

Answer: Option A - Quinidine
Explanation:
Specific pattern of Q-T prolongation is referred to as "Torsades de pointes"


Drugs causing Torsades de Pointes
o Quinidine (most common)
o Sotalol
o Procainamide
o Disopyramide
o Phenothiazines
o Tricyclic antidepressants

Question. 111
Sacubitril is,

A> ACE inhibitor
B> Neutral endopeptidase inhibitor
C> Calcium channel inhibitor
D> Beta adrenergic blocker

Answer: Option B - Neuro-endopeptidase inhibitor
Explanation:
Sacubitril,a prodrug inhibiting neuro-endopeptidase enzyme
Activated to Sacubitril,
Inhibiting enzyme neprilysin (Neutral endopeptidases)
Combination drug used in heart failure patients
Usually combined with ACE inhibitors like valsartan in ratio of 1:1

Question. 112
Niacin therapy is contraindicated in diabetes because

A> Increases the blood sugar levels
B> Causes scleroderma
C> Difficult to give injection
D> Increases the metabolism of oral hypoglycemic drugs

Answer: Option A - increases the blood sugar levels
Explanation:
Niacin therapy has potential effects on blood sugar levels.
Increases the blood glucose levels in diabetes patients

Question. 113
Endothelin acts through which receptors?

A> cAMP
B> cGMP

C> Na+ receptors
D> Calcium receptos

Answer: Option A - cGMP
Explanation:
Endothelin-1 (ET-1) is a potent endogenous vasoconstrictor, mainly secreted by endothelial cells.

Question. 114
Which is the centrally acting alpha 2 agonist muscle relaxant

A> Diazepam
B> Bromocriptine
C> Tizanidine
D> Methocarbamol

Answer: Option -C- Tizanidine
Explanation:
Central alpha 2 adrenergic agonist
Mechanism of action:
Inhibits the release of excitatory amino acids in spinal interneurons
Facilitates the inhibitory transmitter glycine
Inhibits postsynaptic reflexes
Reducing muscle tone, frequency of muscle spasms.without reducing the strength of
muscle.
Indications:
Spasticity in neurological disorders
Painful muscle spasm of spinal origin.
Contraindications:
Patients on antihypertensives specially clonidine.

Question.115
Apixaban is

A> Antithrombin inhibitor
B> Direct X2 inhibitor
C> Platelet activator
D> Clotting Factor XII

Answer: Option B - Direct X2 inhibitor
Explanation:
Direct X2 inhibitor
Anticoagulant for treatment & prophylaxis of venous thromboembolic events
DVT & PE



Question.116
Anaerobes are resistant intrinsically against

A> Beta lactam antibiotics
B> Aminoglycosides
C> Azithromycin
D> Metronidazole

Answer: Option B - Aminoglycosides
Explanation:
Anaerobic bacteria particularly are resistant to aminoglycosides due to lack of oxidative
mechanism to drive drug uptaking process.
Intrinsic resistance / Insensitivity:
Innate ability of bacteria to resist activity of particular antimicrobial agent
Inherent structural or functional characteristics allows for tolerance of a particular drug or
antimicrobial class. i.e., Susceptibility to that particular drug is reduced.
Question. 117
Which is not bacteriostatic antibiotic

A> Clindamycin
B> Vancomycin
C> Tetracycline
D> Cephalosporins

Answer: Option B- Vancomycin
Explanation:
Bacteriostatic antibiotics
Limit bacterial growth by interfering with bacterial protein production, DNA replication, or
other aspects of bacterial cellular metabolism.
Tetracyclines, sulfonamides, clindamycin, spectinomycin,trimethoprim, chloramphenicol,
macrolides and lincosamides.
Bactericidal antibiotics
Inhibit cell wall synthesis (Irreversible killing)
Aminoglycosides. cephalosporins. fluoroquinolones. metronidazole. penicillin. vancomycin

Question. 118
Which of the following causes melanosis coli?

A> Senna
B> Sorbitol
C>Magnesium Sulphate
D> Bisacodyl


Answer: Option A - Senna
Explanation:
Laxative abuse with drugs like senna cause melanosis coli
Anthranoid laxatives (aloe, cascara sagrada, and senna) are derived from naturally occurring
plants
Considered to be stimulant laxatives.
Safer short term use.
Long term abuse can cause melanosis coli & possibly increases risk of colonic cancer.

Question. 119
Which among the following will the choice of antibiotic for a bedridden patient with catheter-related UTI
and pneumonia.

A> Amoxicillin
B> Beta Lactam antibiotics with beta lactamase
C> 3rd gen cephalosporins
D> 2nd gen cephalosporins

Answer: Option B - Beta Lactam Antibiotics
Explanation:
Contains beta lactamase enzyme for potent action against organisms causing UTI

Question. 120
Mycoplasma is resistant to

A> Ceftriaxone
B> Cephalosporins
C> Aminoglycosides
D> Fluoroquinolones

Answer: Option A - Ceftriaxone
Explanation:
Mycoplasma shows resistance towards Ceftriaxone, a third generation cephalosporin (beta lactam
antibiotic)
Lack of cell wall in mycoplasmas makes them intrinsically resistant to -lactams& to all
antimicrobials that target cell wall.
Mycoplasma pneumoniae:
Mycoplasma pneumoniae is a pathogenic mycoplasma responsible for respiratory tract infections in
humans.
First -line treatment: macrolides & related antibiotics, tetracyclines and fluoroquinolones is
preferred.
Question. 121
Tadalafil should not be given with:


A> Vasodilator
B> Antibiotics
C>Vasoconstrictors
D> Valproate

Answer: Option A - Vasodilators
Explanation: Combination with vasodilators results in sudden changes of blood pressure values
Tadalafil relaxes muscles of the blood vessels and increases blood flow to particular areas of the
body.
Used to treat erectile dysfunction (Impotence), and symptoms of benign prostatic hypertrophy
(Enlarged prostate)
Eg: Taking tadalafil with vasodilator drug like nitrate can cause sudden & serious decrease in
blood pressure.
FM
Question 122
Estimate volume of ringer lactate in first 8 hrs for 40% burns in 50 kg male with 2? burns?

A>8 lt
B>4 lt
C>2 lt
D>6 lt
Answer: B
Explanation: Parkland formula most commonly used IV fluid - Lactated Ringer's Solution
Fluid calculation
4 x weight in kg x %TBSA burn
Give 1/2 of that volume in the first 8 hours
Give other 1/2 in next 16 hours
Warning: Despite the formula suggesting cutting the fluid rate in half at 8 hours, the
fluid rate should be gradually reduced throughout the resuscitation to maintain the
targeted urine output, i.e., do not follow the second part of the formula that says to
reduce the rate at 8 hours, adjust the rate based on the urine output.
Example of fluid calculation
50-kg man with 40% TBSA burn
Parkland formula:
4 x 50 x 40 = 8,000 ml
Give 1/2 in first 8 hours = 4,000 ml in first 8 hours
Adjust fluid rate to maintain urine output of 50 ml/hr

Question 123

Posthumous child is:

A>Child delivered after death of biological mother
B>Child delivered after death of biological father
C>Born after death of parents
D>has been abandoned by parents,
Answer: B
Explanation: A child born after death of his/ her biological father
Question 124
M'naghten rule comes under which section of IPC?

A>Crpc 84
B>C pc 48
C>Ipc 84
D>IPC 48
Answer: C
Explanation: M'naghten rule (legal test or right or wrong test) :
It states that an accused person is not legally responsible, if it is clearly proved that at the time of
committing the crime, person was suffering from such a defect of reason from abnormality of mind
that he didn't know the nature and quality of act he was doing or that what he was doing was
wrong i.e. a person is not responsible if he is not of sound mind.
It is accepted in india as law of criminal responsibility and is embodied in section 84 1PC as -
"nothing is an offence which is done by a person, who at the time of doing it, by reason of
unsoundness of mind is incapable of knowing the nature of act, or that he is doing what is either
wrong or contrary to law".
Question 125
Bluish discoloration of gastric mucosa seen in which poisoning?

A>Mercury
B>Cadmium
C>Amytal sodium
D>Arsenic
Answer: C
Explanation:
S. No. Poison
Color
1.
Copper sulfate, amytal capsule
Blue

2.
Ferrous sulfate
Green
3.
Sulphuric hydrochloric/acetic acid
Black/charred
4.
Nitric acid
Yellow
5.
Carbolic acid
Buff/white
6.
Arsenic
White particles
7.
Mercury
Slate
8.
Cresols
Brown
Question 126
Muscle pain, nephropathy caused by which metal poisoning
A>Arsenic
B>Cadmium
C>Mercury
D>Lead
Answer: A
Explanation:
Nephropathy is caused by most of heavy metals.Muscle pain is associated with arsenic.
Question 127
Which is the first organ to putrefy :

A>Brian
B>Heart
C>Prostate
D>Kidney
Answer: A
Explanation: The order of putrefaction is - earliest to last larynx, trachea Stomach, intes ne liver,
spleen Brain, Lungs Heart, Kidney Bladder, Uterus/Prostrate Skin, muscles, tendon lastly,
bones.)
Question 128
Locard is famous for:

A>Theory of exchange
B>Fingerprint study
C>Formula for estimation of stature
D>System of personal identification using the body measurement


Answer: A
Explanation: Edmund locard is famous for theory of exchange.

Question 129
When does basiocciput fuses with basisphenoid?

A>18 to 22
B>22 to 25
C>14-16
D>12-14
Answer: A
Explanation: The basioccipital fuses with the basisphenoid at about 18 to 21 years.
Question 130
What is the smell of mummified body?

A>Odourless
B>Putrid
C>Pungent
D>Offensive

Answer: A
Explanation: A mummy will smell odourless, this is because the internal organs are removed (which are
the biggest factors in the decay process) and replaced with natron (which dries them out, preventing
'proper' decay). The rest of the body is also dried using natron.
Question 131
Patient presented with proximal tubule proteinuria. Which metal is likely to be associated with it?

A>Cadmium
B>Mercury
C>Gold
D>Lead
Answer: A
Explanation: Early kidney damage and proteinuria seen in people, occupationally or environmentally
exposed to cadmium.
Question 132
Which of the following constitutional article is not related to children:

A>23

B>21-A
C>42
D>24
Answer: C
Explanation:
Constitutional Guarantees that are meant specifically for children include:
? Right to free and compulsory elementary education for all children in the 6-14 year age group
(Article 21 A)
? Right to be protected from any hazardous employment till the age of 14 years (Article 24)
? Right to be protected from being abused and forced by economic necessity to enter occupations
unsuited to their age or strength (Article 39(e))
? Right to equal opportunities and facilities to develop in a healthy manner and in conditions of
freedom and dignity and guaranteed protection of childhood and youth against exploitation and
against moral and material abandonment (Article 39 (f))
? Right to early childhood care and education to all children until they complete the age of six years
(Article 45)
Besides, Children also have rights as equal citizens of India, just as any other adult male or female:
? Right to equality (Article 14)
? Right against discrimination (Article 15)
? Right to personal liberty and due process of law (Article 21)
? Right to being protected from being trafficked and forced into bonded labour (Article 23)
? Right of minorities for protection of their interests (Article 29)
? Right of weaker sections of the people to be protected from social injustice and all forms of
exploitation (Article 46)
? Right to nutrition and standard of living and improved public health (Article 47)
Question 133
Which of the statement regarding Factory act is correct

A>Child age less than 14 carrying serious work earn more money
B>Less than 14 yr not done for factory act
C>More than 72 hour work per week
D>More than 82 hours a week
Answer: A
Explanation: The Factory Act prohibits the employment of children below 14 years and declares 15 to 18
years as belonging to the adolescent group. Adolescents require fitness certificate prior to employment in
a job.
Act also prescribed a maximum 48 hours per week, not exceeding 9 Hours per day with at least half hour
rest after 5 hour continuous work.






PSM
Question 134
Incidence of a disease is 4 per 1000 of population with duration of 2 years. Calculate the prevalence?

A>8/1000
B>4/1000
C>2/1000
D>6/1000
Ans. A. 8/1000
Explanation:
Prevalence = (Incidence Rate) x (Average Duration of Disease)
Question 135
Cytotoxic and expired drug disposal is done by which method?

A. Dumping
B. Autoclave
C. Landfill
D. Burning


Ans.C. Landfill
Explanation:
Question 136
For NRR to be 1 couple protection rate should be?

A. 50%
B. 60%
C. 55%


D. 75%
Ans. B. 60%
Explanation:
Couple Protection Rate (CPR)
It is an indicator of the prevalence of contraceptive practice in the community
Definition: the percentage of eligible couples effectively protected against childbirth by one or the
other approved methods of family planning
Sterilization
IUD
Condom
OCP's
NRR = 1 can be achieved only if the CPR > 60%
Question 137
New RNTCP software online to monitor TB control programme is-

A. NIKSHAY
B. NICHAY
C. E- DOTS
D. NIRBHAI


Ans. A.NIKSHAY
Explanation
To keep a track of the TB patients across the country, the Government of India has introduced a
system called NIKSHAY.
The word is combination of two Hindi words NI and KSHAY meaning eradication of tuberculosis.
NIKSHAY (www.nikshay.gov.in) is a web enabled application, which facilitates monitoring of
universal access to TB patients data by all concerned.
The system has been developed jointly by the Central TB Division of the Ministry of Health and
Family Welfare and National Informatics Centre (NIC) and it was launched by the Government of
India in June 2012 with issue of required administrative directions from Central TB Division for use
of NIKSHAY

Question 138
Study unit of ecological study is
A. Population
B. Patient
C. Community
D. Case

Ans. A.Population




Explanation:
In ecological studies the unit of observation is the population or community.
Disease rates and exposures are measured in each of a series of populations and their relation is
examined.
Often the information about disease and exposure is abstracted from published statistics and
therefore does not require expensive or time

Question 139
In a screening test for DM out of 1000 population 90 Were positive.Then gold standard test was done in
which 100 were positive. Calculate the sensitivity?
A. 90/100
B. 100/110
C. 80/100
D. 100/100

Ans. A. 90/100
Explanation:
So, True positive (a)= 90
False negative(c)=10
Sensitivity = a/(a+c)=90/100

Question 140
What is the mass chemoprophylaxis for meningococcal meningitis ?

A. Rifampicin
B. Chloramphenicol
C. Tetracycline


D. Penicillin
Ans. A.Rifampicin
Explanation:
Recommended Chemoprophylaxis for High-Risk Close Contacts:
Age
Dose
Duration
Cautions
RIFAMPICIN:
<1 month
5 mg/kg
Oral every 12 hrs 2 days
>1 month
10 mg/kg
Oral every 12 hrs 2 day
Not recommended for use in pregnancy
CEFTRIAXONE:
<15 years
125mg
IM single dose
>15 years
250 mg
IM single dose
CIPROFLOXACIN
>18 years
500 mg
Oral single dose
Not recommended for use in pregnancy
Question 141
Which among the following is an active form of chlorination?

A. Hypochlorite ion
B. Hydrogen chloride
C. Hypochlorous acid
D. Chloride ion

Ans. C. Hypochlorous acid
Explanation:
The disinfecting action of chlorine is predominantly due to hypochlorous acid.
Hypochlorous acid is most effective form of chlorine and it is almost 70-80 times more effective
than hypochlorite ions.
Question 142
Kala-Azar is found in all endemic areas except.

A. West Bengal
B. UP
C. Bihar
D. Assam






Ans. D. Assam
Explanation:
UP , West Bengal Bihar And Jharkhand are the endemic states of kala Azar
Question 143
Risk among exposed by risk among non exposed is defined to be ?

A. Relative risk
B. Odds ratio
C. Attributable risk
D. None of the above


Ans.A. Relative risk
Explanation:
.
In statistic and epidemiology , relative risk or risk ratio (RR) is the ratio of the probability of an event
occurring (for example, developing a disease, being injured) in an exposed group to the probability of the
event occurring in a comparison, non-exposed group.
Question 144
Pasteurization is done at-

A. 73 oC For 20 min
B. 63 oC For 30 min
C. 72 oC For 30 seconds
D. 63oC For 30 seconds

Ans.B. 63 oC For 30 min
Explanation:
Pasteurization of milk, widely practiced in several countries, notably the United States, requires
temperatures of about 63? C (145? F) maintained for 30 minutes or, alternatively, heating to a
higher temperature, 72? C (162? F), and holding for 15 seconds (and yet higher temperatures for
shorter periods of time).



Question 145
Ideal time gap between 2 live vaccination -
A. 2 weeks
B. 4 weeks
C. 8 weeks
D. 12 weeks

Ans.B.4 weeks
Explanation:
If live parenteral (injected) vaccines (MMR, MMRV, varicella, zoster, and yellow fever) and live intranasal
influenza vaccine (LAIV) are not administered at the same visit, they should be separated by at least 4
weeks.
Question 146
Susceptible person developed disease within range of IP after coming in contact with primary case -
A. Secondary attack rate
B. Case fatality rate
C. Primary attack rate
D. Tertiary attack rate

Ans. A. Secondary attack rate
Explanation:
Secondary Attack Rate (SAR) Number of exposed persons developing the disease within the range of the
incubation period, following exposure to primary case.
Question 147
Out of 100 women who were offered ocp for contraception 10 women got pregnant when followed for
24 months. What is Pearl's index?

A. 10
B. 5
C. 4
D. 2


Ans. B. 5
Explanation:
Pearl Index=10x12x100/100x24= 5
Question 148
Which of the following do not cause hardness of water ?


A. Calcium carbonate
B. Calcium sulphate
C. Calcium bicarbonate
D. Magnesium bicarbonate

Ans. A.Calcium carbonate
Explanation:
Temporary hardness is a type of water hardness caused by the presence of dissolved bicarbonate
minerals (calcium bicarbonate and magnesium bicarbonate).
Permanent hardness is caused by dissolved calcium sulfate (which is not removed by boiling).

Question 149
Which of the following is not an example of direct transmission in communicable diseases ?
A. Transplacental (vertical)
B. Soil
C. Respiratory
D. STD

Ans. C. Respiratory
The modes of transmission of infectious diseases can be classified as:
Direct Transmission. Direct contact; Droplet infection; Contact with soil; Inoculation into skin or
mucosa; Transplacental (vertical)transmission.
Indirect Transmission. Vehicle-borne; Water; food/milk; Vector-borne. Mechanical; Biological.
Airborne.

ENT
Question 150
Water's view is used to obtain diagnostic information of:

A> Maxillary sinus
B> Ethmoidal sinuses
C> Frontal sinus
D> Sphenoid sinus




ANSWER: A
Explanation:
Waters' view (also known as the Occipitomental view) is a radiographic view, where an X-ray beam
is angled at 45? to the orbitomeatal line.
The rays pass from behind the head and are perpendicular to the radiographic plate.
It is commonly used to get a better view of the maxillary sinuses.
Question 151
Tracheostomy indication is:

A> Vocal cord replacement
B> Pharynx replacement
C>Tracheomalacia
D> Foreign body obstructing airway
Answer- D
Explanation:
Indications of tracheostomy:
1- Upper respiratory tract obstruction; Laryngeal, supralaryngeal ,and tracheal causes.( Causes of
stridor )
2- Lower respiratory tract obstruction: (Secretory obstruction, Wet lung syndrome).
Question 152
Which is thickened nerve shown here:











A> Facial Nerve
B> Greater auricular nerve
C> Vagus Nerve
D> Glossopharyngeal Nerve
Answer- B
Explanation:
The greater auricular nerve is a cutaneous branch of the cervical plexus that innervates the skin of the
auricle as well as skin over the parotid gland and mastoid process. The greater auricular nerve also supplies
branches that innervate the deep layer of the parotid fascia.
Origin
The greater auricular nerve arises from the ventral rami of C2 and C3 spinal nerves , although it receives
considerably more fibres from C2.
Course
The greater auricular nerve emerges along the posterior aspect of the sternocleidomastoid muscle at the
punctum nervosum (Erb point) and ascends vertically across the oblique sternocleidomastoid muscle.
When the greater auricular nerve approaches the inferior pole of the parotid gland it divides into anterior
and posterior terminal branches.







Question 153
Caldwell's view is used for:

A> Maxillary sinus
B> Frontal sinus
C> Ethmoidal sinus
D> All of the above
Answer- B
Explanation:
Caldwell's view (or Occipitofrontal view) is a radiographic view of skull, where X-ray plate is angled
at 20? to orbitomeatal line.
The rays pass from behind the head and are perpendicular to radiographic plate.
It is commonly used to get better view of frontal sinuses.

Optha

Question 154
Q) Presbyopic correction in old patients?

A> +1D
B> +2D
C> +3D

D> +4D
OR

Q) Astigmatism in emmetropic eye of elderly person contribute to:

A> +1d
B> +2D
C> +3d
D> +4d
Answer C
Explanation:
Laser vision enhancements
When planning presbyopia-correcting IOL(Intra ocular lens) surgery in a patient with a high level of
pre-existing astigmatism (ie, more than 3 D), a bioptics approach (ie, IOL followed by laser vision
enhancement) may be needed.
LRIs alone are unlikely to correct the astigmatism completely.Limbal Relaxing Incisions (LRI) are a
refractive surgical procedure to correct minor astigmatism in the eye.
There are several different strategies for these planned laser vision enhancements. The first is to
perform the presbyopia-correcting IOL surgery followed by LASIK or PRK.
Question 155
100 day Glaucoma seen in which of the following condition:

A> Central retinal vein occlusion (CRVO)
B> Neovascular glaucoma
C> Central retinal artery occlusion (CRAO)
D> Steroid induced Glaucoma
Answer- A
Explanation:
1. 100 days glaucoma is a neovascular glaucoma occurring in CRVO.
2. It consists of occlusion of central retinal vein without significant retinal ischemia.
3. This results in a venous stasis.
4. Recurrent hemorrhages are frequent and neovascularization of retina and optic disc develop.
5. Retina undergoes pigmentary and atrophic changes.
6. Serious complications are cystoid degeneration of macula, optic atrophy and hemorrhagic or
neovascular glaucoma.
7. Hemorrhagic glaucoma is also known as 100 day glaucoma because it starts 3 months after the
episode of central retinal vein occlusion.

Question 156






Q Roth spots is seen in:
A> Uveal melanoma
B> Acute leukaemia
C> Both a & b
D> None of the above
Answer- B
Explanation:
Roth's spots are retinal hemorrhages with white or pale centers.
Composed of coagulated fibrin including platelets, focal ischemia, inflammatory infiltrate, infectious
organisms, or neoplastic cells.
Roth's spots may be observed in leukemia, diabetes, subacute bacterial endocarditis, pernicious
anemia, ischemic events, hypertensive retinopathy and rarely in HIV retinopathy.
Roth's spots are named after Moritz Roth.

Question 157
Yoke muscle of right lateral rectus:

A> Lt medial rectus
B> Lt superior rectus
C> Lt lateral rectus
D> Lt inferior oblique
Answer - A
Explanation:
Contralaterally paired extraocular muscles that work synergistically to direct the gaze in a given
direction.
For example, in directing the gaze to the right, the right lateral rectus and left medial rectus
operate together as yoke muscles.
Question 158

Severe Conjunctivitis caused by:

A> Neisseria
B> Staphylococcus
C> Streptococcus
D> Haemophilus
ANSWER- A
Explanation:
The most common causes of acute bacterial conjunctivitis are Staphylococcus aureus, Streptococcus
pneumoniae, and Haemophilus influenzae.
Hyperacute cases are usually caused by Neisseria gonorrhoeae or N. meningitidis.
Chronic cases of bacterial conjunctivitis are those lasting longer than 3 weeks, and are typically
caused by Staphylococcus aureus, Moraxella lacunata, or gram-negative enteric flora.
Neisseria gonorrhoeae causes gonococcal conjunctivitis, which usually results from sexual contact
with a person who has a genital infection.
The incidence rates of gonococcal conjunctivitis increase during spring and summer.
This is a potentially devastating ocular infection, because N. gonorrhoeae can cause severe
ulcerative keratitis, which may rapidly progress to corneal perforation.

Question 159
Which is example of the Simple Myopic Astigmatism among the prescriptions given below:

A> Rx (+) sphere
B> Rx will be plano (-)
C> Rx will be (-) sphere
D> (-)(+) (+)(-) on both 90 and 180 degree axis
Answer- B
Explanation:
When eyewear prescriptions are written, they can be classified into different areas depending on
the power or refractive error.
There are seven categories to which prescriptions can fall:
1. Simple Hyperopia, the Rx will be (+) sphere
2. Simple Myopia, the Rx will be (-) sphere
3. Simple Myopic Astigmatism, the Rx will be plano (-)
4. Simple Hyperopic Astigmatism, the Rx will be (+)
5. Compound Hyperopic Astigmatism, major meridian power will be (+) (+) on both 90 and 180
degree axis
6. Compound Myopic Astigmatism, major meridian power will be (-) (-) on both 90 and 180
degree axis
7. Mixed Astigmatism, major meridian powers will be opposites (-)(+) (+)(-) on both 90 and 180
degree axis





Question 160
Blow out fracture of orbit involves:

A> Floor
B> Medial wall
C> Lateral wall
D> Roof
Answer-A
Explanation:
Orbital floor fracture, also known as "blowout" fracture of the orbit.
Blow out fracture of orbit involves:
1. Fractures of the orbital floor are common: it is estimated that about 10% of all facial fractures
are isolated orbital wall fractures (the majority of these being the orbital floor), and that 30-40%
of all facial fractures involve the orbit.
2. The anatomy of the orbital floor predisposes it to fracture.
3. The inferior orbital neurovascular bundle (comprising the infraorbital nerve and artery) courses
within the bony floor of the orbit; the roof of this infraorbital canal is only 0.23mm thick, and
the bone of the posterior medial orbital floor averages 0.37 mm thick.
4. By contrast, the bone of the lateral portion of the orbital floor averages 1.25 mm thick, over 5
times the thickness of the bone over the neurovascular bundle. As one might suspect, it is this
very thin area of the orbital floor overlying the neurovascular bundle where isolated orbital
floor fractures invariably occur.
An orbital blowout fracture of the floor of the left orbit.

Surgery

Question 161.
Which type of retractor is shown in the image






A>Morris retractor
B>Czerny retractor
C>Richardson retractor.
D>Lower lid retractor
Answer: A
Explanation: Used by surgeons and gynecologists so that they can work in the deep layers of the patient.
The Morris retractors are particularly used when abdominal incisions are made as well closed. Making use
of such a retractor is important for the surgeon as it allows them to work with a clear vision and to be able
to move inside the body cavity and repair whatever needs it.
Question 162.
Which of the following statement is true about suture material in the image:
A>Made of rabbit submucosa
B>Made of cat submucosa
C>Not degraded
D>Degraded by enzymatic degradation
Answer: D
Explanation: Sutures such as catgut are derived from sheep submucosa or beef serosa are digested by
proteolytic enzymes in the wound.
Question 163.
Van nuys prognostic index is not based on:

A>Age
B>Microcalcification
C>Size
D>ER status
Answer: D

Explanation: The Van Nuys Prognostic Index is based on size & grade of DCIS, margins and age of patient.
Van Nuys Prognostic Index
Parameter
Score 1
Score 2
Score 3
Van Nuys Classification
Group 1
Group 2
Group 3
Non high nuclear grade Non High nuclear grade High nuclear grade with or
without necrosis
with necrosis
without necrosis
Margins
10mm
1?9mm
<1mm
Size
<15mm
16?40mm
>40mm
Age
>60
40?60
<40


Question 164.
Common cause of chronic pancreatitis
A>Chronic alcohol
B>Chronic pancreatic calculi
C>pancreas divisum
D>Gall bladder stones
Answer: A
Explanation: "Worldwide, alcohol consumption and abuse is associated with chronic pancreatitis in up to
70% of cases "-
Question 165.
The following statement about Keloid is true

A>It contain growth factor
B>Extended excision is the treatment of choice
C>It do not extend beyond the wound
D>None of the above
Answer: A
Explanation: Vaccinations, injections, insect bites, ear piercing, or may arise spontaneously.
Keloids tend to occur 3 months to years after the initial insult, and even minor injuries can result in large
lesions.

They vary in size from a few millimeters to large, pedunculated lesions with a soft to rubbery or hard
consistency.
Although they project above surrounding skin, they rarely extend into underlying subcutaneous tissues.
Certain body sites have a higher incidence of keloid formation, including the skin of the earlobe as well as
the deltoid, presternal, and upper back regions.
They rarely occur on eyelids, genitalia, palms, soles, or across joints.
Keloids rarely involute spontaneously, whereas surgical intervention can lead to recurrence, often with a
worse result.
Question 166
Which of the following layers are cut during fasciotomy ?

A>Skin
B>Skin+subcutaneous fascia
C>Skin+subcutaneous tissue+Superficial fascia
D>Skin+subcutaneous tissue+Superficial fascia+deep fascia
Answer: D
Explanation: Fasciotomy or fasciectomy is a surgical procedure where complete opening of all fascial
envelope is done to relieve tension or pressure commonly to treat the resulting loss of circulation to an
area of tissue or muscle. Fasciotomy is a limb-saving procedure when used to treat acute compartment
syndrome.
Question 167
Which statement is not true regarding crohn's disease :

A> Rectum is not involved
B>Continuous lesion visualized in endoscopy
C>Non caseating granulomas
D>Cobblestone appearance
Answer: B
Explanation: Crohn's disease is frequently associated with "skip lesions," discontinuous areas of active
disease in the colon and small intestine with intervening segments that appear normal.
Question 168
Which is the best investigation for carcinoma head of pancreas:

A> Guided biopsy
B>ERCP
C>Transduodenal/transperitoneal sampling
D> EUS


Answer: A
Explanation: Percutaneous liver biopsy under USG or CT guidance can be diagnostic, but there is a risk of
hemorrhage as these tumors are highly vascular.
Question 169
Abdominal mass is best demonstrated in congenital hypertrophic pyloric stenosis by:

A>In palpation over epigastrium
B>In left hypochondriac
D>Right iliac fossa
C>During feeding
Answer: D
Explanation:
Congenital hypertrophic pyloric stenosis
First born male child is characteristically most commonly affected
It is four times more common in males as in females .
The condition does not present at birth. It is most commonly seen 4 weeks after birth
Presentation
? Vomiting is the presenting symptom (child vomits milk and no bile is present)
? Immediately after vomiting the child is hungry i.e. loss of appetite does not occur.
? Weight loss is striking and rapidly the infant becomes emaciated and dehydrated.
However, greater awareness of pyloric stenosis has led to earlier identification of patients and hence with
fewer incidences of chronic malnutrition and severe dehydration - Nelson 10th/1130
The diagnosis is usually made with a test feed : In this the baby is fed with the bottle by a nurse or
mother and surgeon :
1. palpates the abdomen with a warm hand to detect the lump
2. observes the characteristic peristaltic waves pass across the upper abdomen.
Pathologically musculature of pylorus adjacent to antrum is grossly hypertrophied
Ultrasonography is the investigation of choice
Hyperchloremic alkalosis is common and following diagnosis first concern is to correct metabolic
abnormalities -child is rehydrated with dextrose saline and potassium
Treatment of choice Ramstedt's operation - 'Pyloromyotomy'
Question 170
Calculate GCS of 25 old head injury patient with following parameters confused,opening eyes in
response to pain ,localising pain response to pain
A>6
B>11


C>12
D>7
Answer:B
Explanation:Glasgow coma scale:

Eye opening
Best verbal response
Best motor response
Response
Score
Response
Score
Response
Score
Spontaneously
4
Oriented and Converses 5
Obeys commands
6
To 3
Disoriented
and 4
Localises pain
5
verbal stimuli
converses
To pain
2
Inappropriate words
3
Flexion- withdrawal to 4
pain
Never
1
Incomprehensible
2
Abnormal
flexion 3
words
(decorticate rigidity)

No response
I
Abnormal
extensive 2
obsturing

No response
1
Maximum score is Y : 15
Minimum score is :03
Question 171
RET proto oncogene is associate with development of
A>Medullary carcinoma thyroid

B >Astrocytoma
C >Paraganglioma
D>Hurthle cell tumor thyroid
Answer: A
Explanation: RET proto oncogene is a growth factor receptor (receptor tyrosine kinase)
? The RET protein is a receptor for the glial cell line derived neurotrophic factor and structurally
related proteins that promote cell survival during neural development.
? RET is normally expressed in the following cells
1. Parafollicular C cells of the thyroid
2. Adrenal medulla
3. Parathyroid cell precursors.
? Point mutation in RET extracellular domain which causes constitutive dimerization and activation
leading to:-
1. Medullary thyroid carcinoma
2. Adrenal and parathyroid tumours


Question 172.

Cutoff for surgery in abdominal aortic aneurysm in asymptomatic pts
A>5.5cm
B>6.5cm
C>7.5cm
D>8.5cm

Answer: A
Explanation: Operative repair of the aneurysm with insertion of a prosthetic graft or endovascular
placement of an aortic stent graft is indicated for:
- abdominal aortic aneurysms of any size that are expanding rapidly or are associated with symptoms.
- for asymptomatic aneurysms, abdominal aortic aneurysm repair is indicated if the diameter is >5.5 cm.

Ortho

Question 173.
Muscles affected in De quervain tenosynovitis

A. Abductor pollicis longus and extensor pollicis brevis
B. Adductor pollicis longus and extensor pollicis brevis
C. Abductor pollicis longus and Flexor pollicis brevis
D. Adductor pollicis longus and Flexor pollicis brevis

Ans. A. Abductor pollicis longus and extensor pollicis brevis
Explanation:
De Quervain syndrome, is a tenosynovitis of the sheath or tunnel that surrounds two tendons that
control movement of the thumb.
De Quervain syndrome involves non-inflammatory thickening of the tendons and the synovial
sheaths that the tendons run through.
The two tendons concerned are those of the extensor pollicis brevis and abductor pollicis longus
muscles.
Question 174.
APatient has history of RTA 2 years back, at the same sight he developed pain and swelling.
Xray shows the following features . What will be the diagnosis?






A. Osteogenic sarcoma
B. Ewing's sarcoma
C. Chronic osteomyelitis
D. Multiple myeloma


Ans. C. Chronic osteomyelitis
Explanation:
Garr? sclerosing osteomyelitis, or chronic nonsuppurative sclerosing osteomyelitis, is a form of chronic
osteomyelitis.
Mild inflammation and infection lead to subperiosteal bone deposition.
The disease is frequently asymptomatic.
The characteristic radiographic appearance is an area of periosteal proliferation surrounded by successive
layers of condensed cortical bone (arrows), described as an onion skin appearance.

Question 175.
What is meant byPerilunate dislocations ?

A. Lower radius,scaphoid and lunate and capitate all in same plane
B. Lower radius,scaphoid and capitate in alignment,lunate alone out of plane
C. Lower radius,scaphoid and Lunate in alignment ,capitate alone is out of plane
D. Both lunate and capitate are out of plane

Ans. B.Lower radius,scaphoid and capitate in alignment,lunate alone out of plane
Explanation:





Perilunate dislocation and perilunate fracture dislocation are injuries that involve traumatic rupture
of the radioscaphocapitate (RSC) ligament, the scapholunate interosseous ligament, and the
lunotriquetral interosseous ligament.
Lateral radiographs will reveal loss of collinearity between the capitate, lunate, and radius
Typically the capitate is located dorsal to the lunate and is aligned with the radius
Question 176.
Identify the bone numbered in the X-ray below that most commonly fracture when a person falls on
outstretched hands ?

A. 1
B. 2
C. 3
D. 4

Ans. A.1
Explanation:

A distal radius fracture, also known as wrist fracture, is a break of the part of the radius bone which
is close to the wrist.
Symptoms include pain, bruising, and swelling of rapid onset
The wrist may be deformed.
In younger people these fractures typically occur during sports or a motor vehicle collision.In older
people the most common cause is falling on an outstretched hand.
Specific types include Colles, Smith, Barton, and Hutchinson fractures




Question 176.
Fallen fragment sign

A. Simple bone cyst
B. Osteosarcoma
C. Adamantinoma
D. Aneurysmal bone cyst


Ans. A. Simple bone cyst
Explanation:
The fallen fragment sign refers to the presence of a bone fracture fragment resting dependently in
a cystic bone lesion. T
his finding is said to be pathognomonic for a simple (unicameral) bone cyst following a pathological
fracture.
Although it has occasionally been reported with other cystic lesions, e.g. eosinophilic granuloma
Question 178
You are posted as an intern in causality.Which among the following patients with fracture will be your
1st priority to call ortho PG and inform?

A. Patient's finger is blackening
B. Patient can't extend his arm
C. A 10 cm abrasion
D. Intra articular fracture of Elbow Joint


Ans. A. Patient's finger is blackening
Explanation:
Blackening of finger after Fracture is an indication of cut of of blood supply that may lead to severe
complication like gangrene. So this should be the first Priority to treat among all the above options
Question 179:
In Rheumatoid arthritis, which type of cells are prominently present ?

A. B cells
B. T cells
C. Macrophages


D. Dendritic cells
Ans. B. Macrophages
Explanation:
Synovial lining or intimal layer: Normally, this layer is only 1-3 cells thick. In RA, this lining is greatly
hypertrophied (8-10 cells thick).
Primary cell populations in this layer are fibroblasts and macrophages.
Question 180.
8th and 9th rib costal cartilage forms which type of joint?

A. Costochondral joint
B. Interchondral joint
C. Synovial joint
D. Costovertebral joint


Ans. C. Synovial joint
Explanation:
The sixth, seventh, eighth, ninth and tenth costal cartilages are jointed with each other along the
borders by synovial joints.
Costochondral joint means the joint between the rib and its costal cartilage.
The first costal cartilage of both sides attach to the manubrium sterni. At this joint, no movement is
possible.
The second costal cartilage articulates with the body of sternum and the manubrium sterni by a
synovial joint where movement is possible.
The third to seventh costal cartilages articulate with lateral border of the body of sternum at mobile
synovial joints.
Question 181.
Tom smith septic arthritis is-

A. Acute Gonococcal arthritis
B. Smallpox arthritis
C. Septic arthritis of infancy
D. Chronic pyogenic arthritis


Ans. C. Septic arthritis of infancy
Explanation:
Septic arthritis of infancy (Tom smith septic arthritis)
It is a septic arthritis of hip seen in infants
The onset is acute with rapid abscess formation, which may burst out or be incised and heals
rapidly.
Telescope test is positive
Clinically this condition resembles a congenital dislocation of hip

Pediatrics
Question 182
Which of the fontanelle is the last to close?

A. Anterolateral
B. Anterior
C. Lateral
D. Occipital


Ans. B. Anterior
Explanation:
The posterior fontanelle normally closes 2 to 3 months after birth
The sphenoidal fontanelle is the next to close around 6 months after birth
The mastoid fontanelle closes next from 6 to 18 months after birth;
The anterior fontanelle is generally the last to close between 18?24 months.
Question 183
Which enzyme deficiency causesLesch?Nyhan syndrome ?

A. Hypoxanthine-guanine phosphoribosyltransferase (HGPRT)
B. Xanthine oxidase
C. Adenine phosphoribosyltransferase (APRT)
D. AMP deaminase


Ans. A.Hypoxanthine-guanine phosphoribosyltransferase (HGPRT)
Explanation:
Lesch?Nyhan syndrome (LNS), also known as juvenile gout, is a rare inherited disorder caused by a
deficiency of the enzyme hypoxanthine-guanine phosphoribosyltransferase (HGPRT), produced by
mutations in the HPRT gene located on the X chromosome
Question 184
Which vaccine is not include in indradhanush mission?

A. Tuberculosis
B. Measles
C. Japanese Encephalitis
D. Diphtheria


Ans. C. Japanese Encephalitis
Explanation:
The Mission Indradhanush, depicting seven colours of the rainbow, targets to immunize all children against
seven vaccine preventable diseases namely:
Diphtheria









Pertussis
Tetanus
Childhood Tuberculosis
Polio
Hepatitis B
Measles.
Question 185.
Identify the condition shown in the CT Scan image below.


A. Dandy walker malformation
B. Cerebellar vermis hypoplasia
C. Mega cisterna magna
D. None

Ans. A.Dandy walker malformation
Explanation:

Question 186
Which among the following is the Most common cause for neonatal blindness?

A. Neisseria gonorrhoeae
B. Chlamydia trachomatis
C. Klebsiella
D. Enterobacter

Ans. A. Neisseria gonorrhoeae
Explanation:
Ophthalmia neonatorum (ON) is defined as a purulent conjunctivitis occurring during the first four weeks
of life.
The two most common causative agents are Neisseria gonorrhoeae and Chlamydia trachomatis, the former
being of more concern here because of its propensity to cause blindness.
Obs/Gyne
Question 187
Shoulder dystocia is managed by-

A. Sharp flexion of hip joints towards abdomen
B. Supra pubic pressure
C. 90 degree rotation of posterior shoulder
D. Emergency c-section

Ans. B.Supra pubic pressure
Explanation:
Mnemonic "HELPERR" as a guide for treating shoulder dystocia:
"H" stands for help
"E" stands for evaluate for episiotomy.
"L" stands for legs(pull your legs toward your stomach McRoberts maneuver)
"P" stands for suprapubic pressure
"E" stands for enter maneuvers(internal rotation of baby's shoulders)
"R" stands for remove the posterior arm from the birth canal.
"R" stands for roll the patient.

Question 188
Identify the X ray HSG Shown below :







A. Septate uterus
B. Uterus didelphys
C. Unicornuate uterus
D. Bicornuate uterus


Ans. C. Unicornuate uterus
Explanation :
Question 189
True hermaphroditism karyotype:

A. 45 X0 STREAKED GONADS
B. 46 XX OVO TESTIS
C. 47 XY+9
D. 47 XX

Ans.B.46XX OVO TESTIS
Explanation:
True hermaphrodite or ovotesticular disorder of sexual differentiation (OVO-DSD) is one of the
rarest variety of all inter sex anomalies.
In about 90% of cases, patients have 46 XX karyotype.
Rarely, 46 XY/46 XX mosaicism may occur.
Question 190.
Peripartum cardiomyopathy occurs at-

A. Within 7 days

B. Within 6 weeks
C. Within 24 months
D. Within 5 months


Ans.D. Within 5 months
Explanation:
PPCM is a structural heart muscle disease that occurs in women either at the end of pregnancy or up to
five months after giving birth
Question 191.
Nerve mostly compressed in pregnancy puerperium:
A. Radial nerve
B. Median nerve
C. Femoral nerve
D. Facial nerve



Ans C. Femoral nerve
Explanation:
Obstetricians may consider frequent position changes in labour, avoidance of prolonged hip flexion,
and shortening the pushing time by allowing for passive descent of the fetus before pushing begins
as means of avoiding lateral femoral cutaneous nerve injury
Question 192
In pregnancy which of the following level is altered mostly:
A. TSH
B. Free T3
C. Free T4
D. T3 binding globulin


Ans C.TSH
Explanation:
Increased blood concentrations of T4-binding globulin: TBG is one of several proteins that transport
thyroid hormones in blood, and has the highest affinity for T4 (thyroxine) of the group. Estrogens
stimulate expression of TBG in liver, and the normal rise in estrogen during pregnancy induces
roughly a doubling in serum TBG concentration.
Increased levels of TBG lead to lowered free T4 concentrations, which results in elevated TSH
secretion by the pituitary and, consequently, enhanced production and secretion of thyroid
hormones

Question 193
Paget's is associated with which other cancer:

A. Vulva

B. Vagina
C. Cervix
D. Uterus


Ans. A. Vulva
Explanation:
Extramammary Paget's disease (EMPD), also extramammary Paget disease, is a rare, slow-growing,
usually noninvasive intraepithelial (in the skin) adenocarcinoma outside the mammary gland and
includes Paget's disease of the vulva and the extremely rare Paget's disease of the penis.
Question 194
What is meant by Superfecundation?

A. Fertilization of two or more ova in one intercourse
B. Fertilization of two or more ova in different intercourses in same menstrual cycle
C. Fertilization of ova and then it's division
D. Fertilization of second ovum first being implanted

Ans.B. Two or more ova in different intercourses in same menstrual cycle
Explanation:
Superfecundation is the fertilization of two or more ova from the same cycle by sperm from
separate acts of sexual intercourse, which can lead to twin babies from two separate biological
fathers.
The term superfecundation is derived from fecund, meaning the ability to produce offspring.
Question 195
Fetal heart starts contracting at-

A. 10-12 days
B. 10-12 weeks
C. 3-5 weeks
D. 3- 5 month


Ans. C 3-5 weeks
Explanation:
The fetal heart starts contracting at approximately 23 days of gestation.
Question 196
Anesthesia of choice for cesarean section in severe pre-eclampsia :

A. Spinal
B. GA
C. Epidural
D. Spinal+epidural








Ans. C. Epidural
Explanation:
Continuous Epidural Anesthesia is the first choice for patients with preeclampsia during labour,
Vaginal delivery and cesarean section.
Preeclampsia patient have a risk of severe airway edema , which makes intubation difficult
Continuous Epidural Anesthesia can improve uteroplacental perfusion and also decrease
catecholamine secretions.
Question 197
Which of the following is not a high risk pregnancy?

A. Previous history of manual removal of placenta
B. Anemia
C. Diabetes
D. Obesity



Ans. A. Previous history of manual removal of placenta
Explanation:
Anemia in pregnancy is associated with increased rates of maternal and perinatal mortality,
premature delivery and other adverse outcomes. Hence, identifying anemia predicting risk factors
in high-risk groups such as pregnant women is essential for problem based intervention modalities`
Diabetes is clearly a risk factor for myocardial infarction during pregnancy
Obesity can make a pregnancy more difficult, increasing a woman's chance of developing diabetes
during pregnancy, which can contribute to difficult births.
Question 198
Which of the following is not used in preeclampsia?

A. Methyldopa
B. Atenolol
C. Labetalol
D. Hydralazine


Ans.B. Atenolol
Explanation:


Question 199.
Dilatation & curettage (D&C) is contraindicated in-

A. Pelvic inflammatory disease (PID)
B. Endometriosis
C. Ectopic pregnancy
D. None


Ans. A. Pelvic inflammatory disease (PID)
Explanation:
Predisposing risk factors for PID are:
Sexual contact
History of STI
Procedures involving the upper female genital tract including:
Dilatation & curettage (D&C)
Recent intrauterine device (IUD) insertion
Therapeutic abortion (T/A)
Question 200.
Which of the following is correct regarding placenta?
A. Placental artery provides nutrients through umbilical cord to baby
B. Placenta has Wharton's jelly
C. Placenta has 2 veins and 1 artery
D. Estrogen is secreted by placenta


Ans. B. Placenta has Wharton's jelly
Explanation:
The umbilical cord is a structure that provides vascular flow between the fetus and the placenta.
It contains two arteries and one vein, which are surrounded and supported by gelatinous tissue
known as Wharton's jelly.
Question 201
Acute fatty liver common seen in pregnancy at-

A. 3rd trimester
B. 1st trimester
C. Immediate postpartum
D. Intrapartum


Ans. A. 3rd trimester
Explanation:
Acute fatty liver of pregnancy (AFLP) is a rare, potentially fatal complication that occurs in the third
trimester or early postpartum period.


Question 202
Establishment of fetoplacental circulation seen at-
A. 11 to 13 days
B. 20 to 22 days
C. 7 days
D. 25 to 26 days


Ans. B.20 to 22 days
Explanation:
Important Events Following Fertilization
0' hour
Fertilization (day-15 from LMP)
30 hours
2 cell stage (blastomeres)
40?50 hours
4 cell stage
72 hours
12 cell stage
96 hours
16 cell stage. Morula enters the uterine cavity
5th day
Blastocyst
4?5th day
Zona pellucida disappears
5?6th day
Blastocyst attachment to endometrial surface
6?7th day
Differentiation of cyto and syncytiotrophoblast layers
10th day
Synthesis of hCG by syncytiotrophoblast
9?10th day
Lacunar network forms
10?11th day
Trophoblasts invade endometrial sinusoids establishing uteroplacental
circulation
Interstitial implantation completed with entire decidual coverage
13th day
Primary villi
16th day
Secondary villi
21st day
Tertiary villi
21st?22nd day
Fetal heart. Fetoplacental circulation
Question 203
Fimbriectomy procedure is known as-


A. Uchida method
B. Irving method
C. Madlener technique
D. Kroener method


Ans. D. Kroener method
Explanation:
Uchida technique--A saline solution is injected subserosal in the mid portion of the tube to create a
bleb.
Irving method -- The tube is ligated on either side and mid portion of the tube (between the ties) is
excised.
Madlener technique -It is the easiest method. The loop of the tube is crushed with an artery
forceps.
Kroener method of fimbriectomy is not a common procedure
Question 204
RDA of iodine in lactation in microgram-

A. 150
B. 220
C. 100
D. 250


Ans. D. 250
Explanation:
To accommodate increased iodine needs during pregnancy and lactation, the iodine RDA is 220 mcg/day
for pregnant women and 250 mcg/day for lactating women
Question 205.
Which One of the following is not a cause of secondary Postpartum Haemorrhage?

A. Placenta previa
B. Retained bits of placenta
C. Endometritis
D. Polyp


Ans. A.Placenta previa
Explanation:
Causes of secondary Postpartum Haemorrhage are:
Retained bits of placenta
Postpartum infection
Infection of Cervical and Vaginal Tears
Puerperal Inversion of Uterus
Uterine Polyp or Fibroid:



Undiagnosed carcinoma of cervix
Chorion-epithelioma
Question 206.
Best time to do quadruple test

A. 8-12 weeks
B. 11-15 weeks
C. 15-20 weeks
D. 18-22 weeks


Ans. C.15-20 weeks
Explanation:
The quad screen is done in the second trimester, usually between 15 and 20 weeks of pregnancy.
Ideally, the test should be performed in conjunction with first-trimester screening tests.
Question 207
Drug that is used for fetal lung maturity is:

A. Dexamethasone
B. Folic acid
C. Beclomethasone
D. None


Ans. A. Dexamethasone
Explanation:
Betamethasone and dexamethasone are corticosteroids, also called glucocorticoids, that are given
before birth (antenatally) to speed up a preterm fetuses lung development.
Either is used when a mother is in preterm labor and birth may occur in 24 to 48 hours.
Question 208
In a woman complaining of AUB following image was seen in endoscopic examination of uterus. What
will be the diagnosis?


A. Leiomyoma
B. Adenomyosis


C. Ovarian neoplasm
D. Carcinoma of uterus


Ans. A . Leiomyoma
Explanation:
Leiomyoma is the most common pelvic tumor in women
Benign, originate from myometrial smooth muscles
Symptoms include:
AUB
Pelvic pain and pressure
Infertility or adverse pregnancy outcome
Question 209
In Uterine prolapse how to know if ring is in place?
A. If not expelled after increased abdominal pressure
B. If Bleeding does not occur
C. If patient feels discomfort
D. None


Ans. A.If not expelled after increased abdominal pressure
Explanation:
A vaginal pessary is a removable device placed into the vagina.
It is designed to support areas of pelvic organ prolapse.
A variety of pessaries are available, including the ring pessaries
If not expelled after increased abdominal pressure ring pessary is supposed to be placed in place
Question 210
HT indicated in menopausal women

A. Hot flash
B. Ca breast
C. Endometriosis
D. Uterine bleeding


Ans. A. Hot flash
Explanation:
Hormone Therapy (HT) is one of the government-approved treatments for relief of menopausal
symptoms.
These symptoms, caused by lower levels of estrogen at menopause, include :
Hot flashes,
Sleep disturbances, and
Vaginal dryness.
HT is also approved for the prevention of osteoporosis.


Anaesthesia
Question. 211
Murphy's eye is seen in

A> Macintosh laryngoscope
B> Endotracheal tube
C> LMA
D> Flexible laryngoscope

Answer: Option B - Endotracheal Tube
Explanation:
The "Murphy eye" is the eponymous name for a hole on the side of most endotracheal tubes
(ETTs) that functions as a vent, and prevents the complete obstruction of the patient's airway,
should the primary distal opening of an ETT become occluded

Question. 212
Modified MallamPati grading is used in assessment of

A> Difficult intubation
B> Airway obstruction
C> Death due to aspiration
D> Intubation

Answer: Option A - Difficulty in intubation
Explanation:
Modified Mallampati classification
Class 0: Ability to see any part of the epiglottis upon mouth opening and tongue protrusion
Class I: Soft palate, fauces, uvula, pillars visible
Class II:Soft palate, fauces, uvula visible
Class III: Soft palate, base of uvula visible
Class IV: Soft palate not visible at all
Test: The assessment is performed with the patient sitting up straight, mouth open and tongue
maximally protruded, without speaking or saying "ahh."
Predictive value of modified Mallampati classification
Difficult laryngoscopy:Good accuracy (area under Summary Receiver Operating Characteristic
[SROC] curve 0.89 ? 0.05)
Difficult intubation: Good accuracy (area under SROC curve 0.83 ? 0.03)
Difficult mask ventilation: Poor predictor
Used alone, the Mallampati tests have limited accuracy for predicting the difficult airway and thus
are not useful screening tests
Mallampati classification is only one of 11 nonreassuring findings during airway examination


Question. 213
Which nerve is tested for adequacy of anaesthesia

A> Median Nerve
B> Ulnar Nerve
C> Radial nerve
D> Mandibular nerve

Answer: Option A - Median nerve
Explanation:
Median nerve block can be evaluated by testing the lateral aspect of the ring finger


Question.214
Most effective circuit in spontaneous anaesthesia is

A> Mapleson A
B>Mapleson B
C> Mapleson C
D> Mapleson D

Answer: Option A - Mapleson A
Explanation:
MAPLESON A - (Magill) CIRCUIT
Useful in spontaneous ventilation
The patient inspires whatever is in the tube, using the bag as a volume reservoir.
On eated tube and incoming fresh gas. When the bag is full, exhaled alveolar gas is vented
frxpiration, the bag refills from a combination of expired gas going back up the corrugom
the exhale valve, and then during any expiratory pause, FGF pushes the remaining alveolar
gas out.
Theoretically FGF = 0.7 x Valv should prevent significant rebreathing because deadspace gas
(fresh) is not wasted, but FGF = VA more reliably prevents rebreathing.
Tube volume must exceed (Vt-Vd) or alveolar gas could contaminate the bag.
Inadequate FGF causes rebreathing . Difficult to detect from the CO2 waveform alone - all
that happens is that the rapid fall on inspiration is delayed. If VA exceeds tubing volume,
CO2 enters the bag and will be seen on inspiration on the capnogram.
Controlled ventilation
If the anaesthetist fully closed the valve while squeezing the bag and didn't open it until just
before the bag filled, this circuit would be OK. More commonly the valve is partially closed -
enough to permit adequate tidal volumes despite parallel loss of gas out the valve. FGF must
be increased to compensate for gas lost during inspiration - typically 2.5x minute ventilation.
The Lack system



A co-axial Magill, with the expiratory valve brought coaxially back to the Fresh Gas outlet.
Not popular due to inefficiency during controlled ventilation.


Question. 215
What is mechanism of action of Curanium drugs as muscle relaxant?

A> Persistently depolarizing at Neuromuscular junction
B> Act competitively on Ach receptors blocking post-synaptically
C> Repetitive stimulation of Ach receptors on muscle end plate
D> Inhibiting the calcium channel on presynaptic membrane

Answer: Option B - Act competitively on Ach receptors blocking post-synaptically
Explanation:
Candocuronium iodide is an aminosteroid neuromuscular-blocking drug or skeletal muscle
relaxant in the category of non-depolarizing neuromuscular-blocking drugs.
Acts on Ach receptors competitively post-synaptically blocking them.
Potential adjunctive use in anaesthesia to facilitate endotracheal intubation & provide skeletal
muscle relaxation.
Candocuronium demonstrated a short duration and a rapid onset of action, with little or no
ganglion blocking activity, and it was only slightly less potent than pancuronium
Skin
Question 216
Identify the following lesion.

A. Becker nevus
B. Hypopigmented macule
C. Spitz nevus.
D. Epidermal nevus


Ans. A. Becker nevus
A Becker nevus (nevus in American spelling) is a late-onset epidermal nevus or birthmark occurring
mostly in males. It is also known as Becker melanosis.



It is due to an overgrowth of the epidermis (upper layers of the skin), pigment cells (melanocytes)
and hair follicles.
Question 217
Cutis marmorata occurs due to exposure to ?

A. Cold temperature
B. Dust
C. Hot temperature
D. Humidity


Ans. A.Cold temperature
Explanation:
Cutis marmorata is a condition where in the skin has a pinkish blue mottled or marbled appearance
when subjected to cold temperatures.
It is seen throughout infancy and in 50 % of children.
Rewarming restores the skin to normal.
It is caused by superficial small blood vessels in the skin dilating and contracting at the same time .
Question 218
:
A child has a rash as shown in the picture .His family history is positive for asthma . What could be the
most probable diagnosis ?

A. Seborrheic dermatitis
B. Atopic dermatitis
C. Allergic contact dermatitis
D. Erysipelas


Ans. C. Allergic contact dermatitis
Explanation:
ACD is a form of contact dermatitis that manifests as an allergic response caused by contact with a
substance .Its a hypersensitive reaction characterised by the presence of rash or a skin lesion in the form of
papules , blisters or vesicles etc . The differential features are
It is confined to the area where the trigger touched the zone,

It occurs after a day or two of the exposure and
The symptoms reappear when in contact with the allergen .
Radio
Question 219
Dye used in diagnosis of esophageal perforation:

A> Iohexol
B> Barium sulphate
C> Gadolinium
D> Iodine dye

Answer-B
Explanation:
Barium sulfate in suspension is frequently used medically as a radiocontrast agent for X-ray
imaging and other diagnostic procedures.
It is most often used in imaging of the GI tract during what is colloquially known as a "barium meal".
Fluoroscopy
most sensitive within the first 24 hours.
patient examined semi-supine (~20 degrees) on fluoroscopy table
a water-soluble agent should be used initially as barium can cause mediastinitis
esophageal perforation may be represented as mucosal irregularity or gross extraluminal contrast
extravasation
some authors suggest the use of small amounts of low or high concentrations of barium if no leak is
evident on initial screening with water soluble contrast

Iohexol, trade names Omnipaque among others, is a contrast agent used during X-rays.This includes when
visualizing arteries, veins, ventricles of the brain, the urinary system, and joints, as well as during computer
tomography. It is given by mouth, injection into a vein, or into a body cavity.
Question 220
Bragg peak effect pronounced in:

A> X ray
B> Proton
C> Neutron
D> Electron
Answer- B
Explanation:




Bragg peak
Pronounced peak on the Bragg curve which plots the energy loss ofionizing radiation during its
travel through matter.
Forprotons,-rays, and otherion rays, the peak occurs immediately before the particles come to
rest. This is called Bragg peak, afterWilliam Henry Bragg who discovered it in 1903.
Bragg curve
Typical for heavy charged particles and describes energy loss of ionizing radiation during travel
through matter.
For this curve is typical the Bragg peak, which is the result of 1/v2 dependency ofthe stopping
power. This peak occurs because thecross section of interaction increases immediately before the
particle come to rest. For most of the track, the charge remains unchanged and the specific energy
loss increases according to the 1/v2.
Bragg Curve is typical for heavy charged particles and plots the energy loss during its travel through matter.

Question 221
Salt and Pepper pot appearance of skull seen in:

A> Hyperparathyroidism
B> Multiple myeloma
C> Hyperthyroidism
D> Pseudo hyperparathyroidism
Answer- A
Explanation:
Pepperpot skull is occasionally used in place of salt and pepper skull to describe the typical
radiographic appearance of multiple small radiolucent lesions of the skull vault.
In primary hyperparathyroidism, extensive resorption bone in the skull in combination with cystic
areas of osteopenia are termed pepper pot skull.
Classically seen in hyperparathyroidism, and is occasionally used (inaccurately) to describe the
raindrop skull of multiple myeloma.





Question 222
Imaging techniques used in Uterus anomalies EXCEPT:

A> HSG
B> MRI guided HSG
C> CT guided HSG
D> USG

Answer C
Explanation:
Imaging studies, such as a hysterosalpingogram (HSG) and ultrasound, or an MRI are required to
visualise the uterus and confirm that a congenital uterine anomaly is present.
A hysterosalpingogram is not considered as useful due to the inability of the technique to evaluate
the exterior contour of the uterus and distinguish between a bicornuate and septate uterus.
In addition,laparoscopy and/orhysteroscopy may be indicated.
Question 223
Spot radiograph from a double contrast esophagram. Image represents:



A> Esophageal atresia
B> Esophageal stenosis
C> Feline oesophagus
D> Tracheoesophageal fistula
Answer- C
Explanation:
In given image, there are numerous 1-2 mm radiolucent folds across the oesophagus. The folds are angled
with respect to the center of the oesophagus in a "herringbone" pattern.
The folds occur transiently.
Feline oesophagus also known as oesophageal shiver, refers to the transient transverse bands seen in the
mid and lower oesophagus on a double contrast barium swallow.
The appearance is almost always associated with active gastro-oesophageal refluxand is thought to be due
to contraction of the muscularis mucosae with resultant shortening of the oesophagus and 'bunching up' of
the mucosa in the lumen.
Radiographic features
The folds are 1-2 mm thick and run horizontally around the entire circumference of the oseophageal
lumen. The findings are transient, seen following reflux and not during swallowing. The appearance is
confined to the distal two-thirds of the thoracic oesophagus.
Question 224
MRI of skull represents:







A> Vein of Galen
B> Dandy walker Syndrome
C> Pneumocephalus
D> Crouzon syndrome
Answer A
Explanation:
The vein of Galen is located under the cerebral hemispheres and drains the anterior and central regions of
the brain into the sinuses of the posterior cerebral fossa.
The vein of Galen, also known as the great cerebral vein or great vein of Galen, is a short trunk formed by
the union of the two internal cerebral veins and basal veins of Rosenthal. It lies in the quadrigeminal
cistern. It curves backward and upward around the posterior border of the splenium of the corpus
callosum to drain into the confluence of the inferior sagittal sinus and the anterior extremity of the straight
sinus.


Question 225
What is diagnosis based on given image:










A> Uterus didelphys
B> Bicornuate Uterus
C> Unicornuate Uterus
D>Septate uterus
Answer-C
Explanation:
A unicornuate uterus or unicornis unicollis is a type of Mullerian duct anomaly (class II) that is the
second most commonly associated with miscarriages.
This type can account for ~10% (range 6-13%) of uterine anomalies and infertility is seen in ~12.5%
(range 5-20%) of cases.
Above image unicornuate uterus as seen on a hysterosalpingogram represents:
The endometrial cavity usually assumes a fusiform (banana type) shape (except for type a where there may
a small cavitation filling defect), tapering at the apex and draining into a single fallopian tube. The uterus is
generally shifted off the midline.
Question 226
X ray of skull showing which lesions in brain:
A> Paget's disease



B> Multiple myeloma
C> Osteosarcoma
D> Osteomyelitis
Answer- A
Explanation:
Paget diseaseof the bone is a common, chronic bone disorder characterised by excessive abnormal bone
remodelling. It frequently affects the pelvis, spine, skull and proximal long bones and has characteristic
radiographic features.
Radiographic features
The early phase features osteolytic (lucent) region which is later followed by coarsened trabeculae and
bony enlargement. Sclerotic changes occur much later in the disease process.
osteoporosis circumscripta: large, well-defined lytic lesion
cotton wool appearance: mixed lytic and sclerotic lesions of the skull
diploic widening: both inner and outer calvarial tables are involved, with the former usually more
extensively affected
Tam o'Shanter sign: frontal bone enlargement, with the appearance of the skull falling over the
facial bones, like a Tam o' Shanter hat.
( Thickened diploe of the skull and ill-defined 'fluffy' sclerotic areas most pronounced in the frontal bone.)
Question 227
CT of Thorax represents:










A> Ascending Aortic dissection
B> Descending Aortic dissection
C> Aortic aneurysm
D> Cystic fibrosis

Answer B
Explanation:
Aortic dissection (AD) occurs when an injury to the innermost layer of the aorta allows blood to flow
between the layers of the aortic wall, forcing the layers apart.
The diagnosis of aortic dissection is based on clinical suspicion combined with imaging studies. Chest x-ray
will sometimes show a "widened mediastinum". This occurs because the enlarged aorta casts a larger
shadow on the x-ray detector. If this is seen, and there is a high clinical suspicion of a dissection, a CT scan
of the chest with intravenous contrast is ordered (see image).
The CT scan will show the true and false lumens associated with dissection.

Question 228
CT scan of abdomen showing an area that branching into the liver. Identify the structure ?











A> SVC
B> IVC
C> Portal vein
D> Splenic vein
Answer- C
Explanation:
The portal vein or hepatic portal vein is a blood vessel that carries blood from the gastrointestinal
tract, gallbladder, pancreas and spleen to the liver.
This blood contains nutrients and toxins extracted from digested contents. Approximately 75% of
total liver blood flow is through the portal vein, with the remainder coming from the hepatic artery
proper. The blood leaves the liver to the heart in the hepatic veins.


Question 229
Identify artery `X' in the given angiography anatomy image:











A> Superior mesenteric artery
B> Subclavian artery
C> Celiac artery
D> Brachiocephalic artery
Answer ? A
Explanation:
The superior mesenteric artery (SMA) is a major artery of the abdomen. It arises from the abdominal aorta,
and supplies arterial blood to the organs of the midgut ? which spans from the major duodenal papilla (of
the duodenum) to the proximal 2/3 of the transverse colon.
Anatomical Position
The superior mesenteric artery is the second of the three major anterior branches of the abdominal aorta
(the other two are the coeliac trunk and inferior mesenteric artery). It arises anteriorly from the abdominal
aorta at the level of the L1 vertebrae, immediately inferior to the origin of the coeliac trunk.
After arising from the abdominal aorta, the superior mesenteric artery descends down the posterior aspect
of the abdomen. At this point, it has several important anatomical relations:
Anterior to the SMA ? pyloric part of the stomach, splenic vein and neck of the pancreas.
Posterior to the SMA ? left renal vein, uncinate process of the pancreas and inferior part of the
duodenum.
The uncinate process is the only part of the pancreas that hooks around the back of the
SMA.
Major Branches
The superior mesenteric artery then gives rise to various branches that supply the small
intestines, cecum, ascending and part of the transverse colon (fig).








The superior mesenteric artery and its branches.

Psychiatry

Question. 230
Semen squeeze

A>Erectile dysfunction
B> Premature ejaculation
C> Retrograde ejaculation
D> Antegrade ejaculation

Answer: Option B- Premature Ejaculation
Explanation:
Squeeze Technique:
Variation of the Masters and Johnson method.
As a man approaches climax, either he or his partner squeezes the tip of the penis just
below the head of the penis as he approaches the point of climax.
Pressure is held there until the sensation of impending orgasm diminishes.
This pressure can even be held until there is some reduction in erection.
The process can then be started over again so that over time a man prolongs the time
period until he reaches ejaculation.
Useful to treat cases of premature ejaculation



Question. 231.

A patient with a history of RTA before 2 months presents with complaints of dreams of accidents. He is
able to visualize the same scene whenever he visits the place. Hence is afraid to go back to the accident
site. Identify the type of disorder that he might be suffering from?

A> Adjustment disorder
B> PTSD
C> Anxiety disorder
D> OCD

Answer: Option B - Post-traumatic Stress disorder
Explanation:
Post-traumatic stress disorder, basically a type of anxiety disorder
Though it shares features with other anxiety disorders, has its own specific characteristics in
presentation.
Criteria 1:
Etiologically significant trauma should be present to be classified under this condition.
Criteria 2:
"Intentionality" or "aboutness", an important factor in PTSD.
PTSD concerns with memory intrusion of past stressors into present.
Nightmares, flashbacks, or reliving experiences should be related to the past experience.
Criteria 3:
Avoid a stimulus or activity that provokes the memory of past event.

Question. 232
Freud's theory of dream includes all except:

A> Displacement
B> Condensation
C> Symbolisation
D> Correlation

Answer: Option -D - Correlation
Explanation:
Sigmund Freud theory of dream:
Dream work involves the process of condensation, displacement, and secondary elaboration.
Concept of unconscious mind:
Primary assumption of Freudian theory is that the unconscious mind governs behavior to a
greater degree than people suspect.
Goal of psychoanalysis is to make the unconscious, conscious.
Theory of dream:
Freud theory of dream elaborates the state of unconscious mind with respect to dream
interpretation,
Process involved include condensation, displacement, and secondary elaboration.
Displacement takes place when we transform the person or object we are really concerned
about to someone else.

The process of condensation is the joining of two or more ideas/images into one.
Question. 233
Expression and consequent release of previously repressed emotion is called as

A> Regression
B> Dissociation
C> Abreaction
D> All of the above

Answer: Option C - Abreaction
Explanation:
The expression and consequent release of a previously repressed emotion, achieved through
reliving the experience that caused it.
Done typically through
Hypnosis
Suggestion

Question.234
All are habit disorder except

A> Nail biting
B> Thumb sucking
C> Temper tantrum
D> Tics

Answer: Option C- Temper tantrum
Explanation:
Definition:
Habit disorder is a term used to describe several related disorders linked by the presence of
repetitive and relatively stable behaviour that seem to occur beyond the awareness of the
person performing the behaviour.
The first group of habit disorder includes "Tic Disorder"
Tics are involuntary movements, sounds, or words that are sudden, rapid, recurrent and non-
rhythmic"
In addition to TD's, body-focussed behaviours,
Recurrent hair pulling - Trichotillomania (TTM)
Skin picking (SP)
Nail biting , are included within habit disorders.
Head banging, rocking of body, teeth grinding & thumb sucking are repetitive disorders.
Seen in children between age group of 6 months to 2 years.
Benign & self-limited.
The movements serve a means of tension discharging in children.
As children become older, they learn to inhibit some of their rhythmic patterns.
Undue attention by parents can lead to aggravation of these problems.


Question. 235
New name of mental retardation according to American Association of Mental Retardation

A> Feeble Mindedness
B> Madness
C> Intellectual disability
D> Mentally unstable

Answer: Option C - Intellectual disability
Explanation:
Intellectual disability (ID), also known as general learning disability,
Mental retardation (MR), is a generalized neurodevelopmental disorder characterized by
significantly impaired intellectual and adaptive functioning.

Question. 236
Now-a-days Down syndrome Is referred to as.

A> Submental disorder
B> Oligophrenia
C> Madness
D> Mentally unstable

Answer: Option A - Submental disorder
Explanation:
Majority of children with Down syndrome function in mild to moderate range of mental
retardation.
Down syndrome / Trisomy 21 -
Genetic disorder caused by presence of all or part of a third copy ofchromosome 21.
Typically associated with physical growth delays, characteristic facial features and mild to
moderate intellectual disability.


Misc Questions

Question 237
A 55 years aged chronic alcoholic male, presented with irrelevant talks, tremor and sweating. He had his
last drink 3 days back. What will the probable diagnosis?

A> Delirium tremens
B> Korsakoff psychosis
C> Post-Acute withdrawal syndrome
D> Discontinuation syndrome




Answer: Option A - Delirium Tremens
Explanation:
The description of symptoms is related to the condition "Delirium Tremens"
Delirium Tremens / Alcohol Withdrawal Delirium (AWD):
Most severe form of ethanol withdrawal manifested by,
Altered mental status (Global confusion)
Autonomic hyperactivity (Sympathetic overdrive)
Mechanism:
Alcohol abuse affects neurotransmitter systems in brain mainly by,
Loss of GABA inhibitory mechanism - Reduces chloride ion influx.

Alcohol acts as NMDA receptor antagonist - Withdrawal increases the excitatory
neurotransmitter.
The clinical manifestations of ethanol withdrawal are combination effects of GABA & NMDA
receptor activity.
Thus causing tremors, diaphoresis, tachycardia, anxiety & in severe cases Seizure.

Question 238.
Which of the following is carrying agent for the disease with given characteristic on polarized
microscopy?
A> Anopheles
B>Ixodes scapularis ticks
C> Louse
D> Rat flea

Answer: B
Explanation:
Babesiosis infects the RBCs and resides inside the RBCs ( intraerythrocytic).
Intraerythrocytic infection of Babesiosis is characterised by maltese cross.
Maltese cross is a characteristic arrangement of parasites within the erythrocytes --->Parasites
within erythrocytes are arranged such that pointed ends of four parasites come in contact thereby
giving a tetrad configuration resembling a maltese cross.
Tetrad forms or 'Maltese cross' appearance is considered pathognomic of Babesiosis.
Babesiosis can easily be confused with P. falciparum malaria.Following two features distinguish
Babesiosis from malaria
1. Presence of maltese cross in Babesiosis ( absent in malaria)

2. Absence of pigment Hemozoin in Babesiosis ( present in malaria)

Note - Maltese cross is also seen in cryptococcus and aspergillus.
Question 239.
Which of the following is best Stent for Femoropopliteal Bypass?

A>Dacron
B>Reversed saphenous
C>PTFE
D>None

Answer: C- PTFE-covered self-expanding nitinol stents
Explanation:
PTFE-covered stents are engineered with a 30?100 micron pore size to allow for endothelial lining
of the stent-graft and vessel healing.
Only self-expanding covered nitinol stents should be used in femoropopliteal interventions.
Question 240
Trilene is degraded by:

A>Enzymatic Degradation
B>Non Enzymatic degradation
C>Chemical Degradation
D>None
Answer:A
Explanation:
Trilene or trichloroethylene is a good analgesic, less depressant, and non-flammable.
Cardiac dysrhythmia, or tachypnoea may occur during administration.
It should not be used in the closed circuit as it reacts with soda-lime to produce a toxic
gas(phosgene).
Recovery is slow and nausea as well as vomiting may be present.
It should not be used with adrenaline infiltration lest dysrhythmia be converted to ventricular
fibrillation.
Degradation:
Brought about by enzymatic degradation
The enzyme that starts one branch of this pathway, toluene 1,2-dioxygenase, has many other
catalytic abilities, which are documented in a table of the Reactions of Toluene 1,2-Dioxygenase.
The spontaneous degradation of trichloroethylene epoxide can produce as many as four products:
dichloroacetate, carbon monoxide, glyoxylate, and formate. The number, type, and proportion of
products seen depends on the local environment.
Question 241.

The earliest feature of 3rd cranial nerve involvement in diabetes mellitus patient is
A>Normal light reflex
B>Abnormal light reflex
C>Normal light and accommodation reflex
D>Abnormal light and accommodation reflex

Answer:A
Explanation: The oculomotor nerve is the third cranial nerve. It enters the orbit via the superior orbital
fissure and innervates muscles that enable most movements of the eye and that raise the eyelid. The nerve
also contains fibers that innervate the muscles that enable pupillary constriction and accommodation
(ability to focus on near objects as in reading). The oculomotor nerve is derived from the basal plate of the
embryonic midbrain.In people with diabetes and older than 50 years of age, an oculomotor nerve palsy
occurs.

Question 242
During squint surgery, anesthesiologist sees the machine and see the bp suddenly drops to 40. What will
be best immediate management

A>Give atropine
B>Increase level of anesthesia
C>Ask the surgeon to stop the surgery
D>Give adrenaline
Answer: D
Explantation: adrenaline should be given to raise the blood pressure.
Epinephrine, also known as adrenalin or adrenaline, is a hormone, neurotransmitter, and medication.
Epinephrine is normally produced by both the adrenal glands and certain neurons.
It plays an important role in the fight-or-flight response by increasing blood flow to muscles, output of
the heart, pupil dilation, and blood sugar. It does this by binding to alpha and beta receptors.

Physiologic responses to epinephrine by organ
Organ
Effects
Heart
Increases heart rate; contractility; conduction across AV node
Lungs
Increases respiratory rate; bronchodilation
Systemic
Vasoconstriction and vasodilation
Liver
Stimulates glycogenolysis
Systemic
Triggers lipolysis
Systemic
Muscle contraction


Question 243
All are special visceral efferent column except

A>Glossopharyngeal n
B>Nucleus ambiguus
C>vagus nerve
D>trigeminal nerve
Answer: B
Explanation: Special visceral efferent fibers (SVE) are the efferent nerve fibers that provide motor
innervation to the muscles of the pharyngeal arches in humans,The only nerves containing SVE fibers are
cranial nerves: the trigeminal nerve (V), the facial nerve (VII), the glossopharyngeal nerve (IX), the vagus
nerve (X) and the accessory nerve
Question 244.
Which of the following condition is NOT caused by Parvovirus B19?

A>Roseola infantum
B>Aplastic anemia in sickle cell disease
C>Fetal hydrops
D>Erythema infectiosum
Answer: A
Explanation:
Primary infection by parvovirus B19 often produces an acute, severe, and sometimes fatal anemia
manifested as a rapid fall in red blood cell count and hemoglobin.
These patients may present initially with no clinical symptoms other than fever; this is commonly referred
to as aplastic crisis.
Erythema infectiosum (also referred to as fifth disease or academy rash) is a more common disease that is
clearly attributable to parvovirus B19.
Active transplacental transmission of parvovirus B19 can occur during primary infections in the first 20
weeks of pregnancy, sometimes resulting in stillbirth of fetuses that are profoundly anemic.
The progress can be so severe that hypoxic damage to the heart, liver, and other tissues leads to extensive
edema (hydrops fetalis).
Question 245
Which of the following statements is not true about iliolumbar ligament?

A> Upper fibres attached to iliac crest
B> Lower fibres attached to base of sacrum
C>Help in maintaining lumbosacral joint stability
D>Upper attachment to transverse process of T12


Answer: Option D - Upper attachment to transverse process of T12
Explanation: The ligament attaches to T5

Iliolumbar ligament:
Strong ligament passing from the tip of transverse process of fifth lumbar vertebra to posterior
part of inner lip of iliac crest
Upper bands gets attached to the iliac crest.
Lower bands gets attached to base of sacrum.
Major function is to strengthen the lumbosacral joint.
Question 246.
Where will be the placement location for Auditory Brainstem Implant?

A>Scala tympani
B>Recess of 4th ventricle
C>IAC
D>back of ear
Answer: Option B - Recess of 4th ventricle.
Explanation: The implant is usually placed in the lateral recess of the fourth ventricle at the time of
tumor resection to stimulate the cochlear nucleus
Auditory Brainstem Implant (ABI):
Tumor resection surgery in NF patients result in cochlear nerve damage or loss of function of nerve
resulting in deafness.
ABI are useful in restoring auditory perception to deaf patients with neurofibromatosis type 2 (NF2)
Also used in treatment of congenitally deaf children with cochlear malformations or cochlear nerve
deficiencies.
Placement location: Lateral recess of 4th ventricle
Question 247
Which condition is associated with Congenital adrenal hypoplasia?

A> Male pseudohermaphroditism
B> Female pseudohermaphroditism
C> True pseudohermaphroditism
D> Sequential pseudohermaphroditism
Answer - Option A - Male pseudohermaphroditism
Explanation:
X-linked congenital adrenal hypoplasia (AHC) is a rare developmental disorder of the human
adrenal cortex and is caused by deletion or mutation of the DAX-1 gene.
DAX1 necessary for differentiation of the definitive adult adrenal cortex

Male pseudohermaphroditism results from inadequate androgen secretion or inappropriate
androgen action
Question 248
Which is true regarding ataxia telangiectasia:

A>Increase in AFP
B>Increases the risk of squamous cell carcinoma
C>Autosomal dominant
D> None of above
Answer: Option A - Increase in AFP
Explanation: Increase in alpha-fetoprotein is observed in Ataxia telangiectasia

Ataxia-telangiectasia / Ataxia-telangiectasia syndrome / Louis-Bar syndrome
Rare, neurodegenerative, autosomal recessive disorder causing severe disability.
Ataxia refers to poor coordination; Telangiectasia refers to small dilated blood vessels.
Parts affected:
Cerebellum - movement & coordination difficulties
Immune system - Predisposing to infections.
Genetic repair system - Preventing process for repairing DNA - Cancer risk
Features:
Increased incidence of lymphoma & Leukemia
Increased alpha-Fetoprotein levels
Oculomotor apraxia (difficulty in coordination between head & eye movements)
Dysarthria

Question 249
A diabetic patient 2 days after post cataract surgery develops develops hypopyon. What will be the
management?

A. Intravitreal antibiotics
B. Eye drops
C. Surgery
D. No treatment required


Ans. A. Intravitreal antibiotics
Explanation:
Question 250
What is the Thinnest part of neuro-retinal rim according to ISNT rule?
A. Inferior

B. Superficial
C. Temporal
D. Medial


Ans. C. Temporal
Explanation:
The ISNT rule is an easy way to remember how the optic nerve is supposed to look in a normal eye.
Normally the neuro-retinal rim is thickest Inferiorly and thinnest Temporally. With glaucoma, however, you
begin to see vertical thinning, with atrophy along the inferior and superior rims.
Question: 251
Leiden thrombophilia is caused by mutational deficiency of which of the following factors?

A. Factor V
B. Factor VII
C. Factor IX
D. Factor X


Ans. A. Factor V
Explanation:
Factor V Leiden thrombophilia is an inherited disorder of blood clotting. Factor V Leiden is the name of a
specific mutation (genetic alteration) that results in thrombophilia, or an increased tendency to form
abnormal blood clots in blood vessels. Factor V Leiden is the most common inherited form of
thrombophilia.
Question:252
Anteversion of uterus is maintained by?

A. Cardinal
B. Uterosacral
C. Pubocervical
D. Round


Ans. D. Round
Explanation:
In most women, the uterus is anteverted and anteflexed. The function of the round ligament is
maintenance of the anteversion of the uterus(a position where the fundus of the uterus is turned forward
at the junction of cervix and vagina) during pregnancy. Normally, the cardinal ligament is what supports
the uterine angle (angle of anteversion).
Question:253
Long standing pelvic inflammation may lead to which of the following conditions?

A. Pyometra


B. Uterine polyposis
C. Pseudopregnancy
D. Cystic endometrial hyperplasia


Ans. A.Pyometra
Explanation:
Pyometra is collection of pus due to obstruction of flow in the uterine cavity.
It may be due to Long standing PID or secondary to cervical stenosis.
Question:254
A red soft to firm swelling on sternum that on biopsy shows following histology. What is the diagnosis?


A. Hemangioma
B. Osteochondroma
C. Osteoid osteoma
D. Paget disease

Ans. A. Hemangioma
Explanation:
Clinical presentation
These tumours are slow growing and are generally asymptomatic unless they exert mass effect on sensitive
structures. Occasionally they may present as a swelling or a palpable mass, especially in the skull. When
large and strategically located they may present with a pathological fracture.
If they are high-flow lesions, shunt-related symptoms may also be present.
Pathology
Primary intraosseous haemangiomas are slow growing vascular neoplasms, usually located in the
medullary cavity. They are classified as benign, but rarely may be locally aggressive.
Histology
Histologically, intraosseous haemangiomas demonstrate hamartomatous vascular tissue within
endothelium, but may also contain fat, smooth muscle, fibrous tissue, and thrombi.
Question:255
What differentiates delirium from dementia?
A. Confusion

B. Difficulty in communicating
C. Hallucination
D. Sudden change


Answer:D
Explanation:
Delirium
Also called the acute confusional state, delirium is a medical condition that results in confusion and other
disruptions in thinking and behavior, including changes in perception, attention, mood and activity level.
In dementia, changes in memory and intellect are slowly evident over months or years. Delirium is a more
abrupt confusion, emerging over days or weeks, and represents a suddenchange from the person's
previous course of dementia. Thinking becomes more disorganized, and maintaining a coherent
conversation may not be possible.
The hallmark separating delirium from underlying dementia is inattention. The individual simply cannot
focus on one idea or task.

Question:256
Genital warts are caused by which virus?
A. Herpes simplex
B. Human papilloma
C. Cytomegalovirus
D. Varicella zoster

Ans. B.Human papilloma
Explanation:
Genital warts are soft growths that appear on the genitals. Genital warts are a sexually transmitted
infection (STI) caused by certain strains of the human papillomavirus (HPV). These skin growths can cause
pain, discomfort, and itching.
Question:257
Which drug regimen is given in a pregnant woman with HIV infection?
A. Tenofovir disoproxil fumarate with emtricitabine
B. Tenofovir disoproxil fumarate with lamivudine
C. Abacavir with lamivudine
D. All

Ans. D. All
Explanation: Preferred Regimens for HIV Antiretroviral Therapy (ART) in Pregnancy
Two-NRTI backbone
Regimens include the following:

Tenofovir disoproxil fumarate with emtricitabine (TDF/FTC co-formulated) or tenofovir
disoproxil fumarate with lamivudine (3TC) once daily (use with caution in renal insufficiency)
or
Abacavir with lamivudine (ABC/3TC) once daily (only if HLA-B5701?negative); avoid
combination with ritonavir-boosted atazanavir if the pretreatment HIV viral load exceeds
100,000 copies/mL.
For women who have never taken HIV medicines, the preferred HIV regimen should include two
nucleoside reverse transcriptase inhibitors (NRTIs) plus an integrase strand transfer inhibitor (INSTI), a
non-nucleoside reverse transcriptase inhibitor (NNRTI), or a protease inhibitor (PI) with low-dose
ritonavir (brand name: Norvir).
The regimen generally should include at least one of the following NRTIs that pass easily across the
placenta:
abacavir (brand name: Ziagen)
emtricitabine (brand name: Emtriva)
lamivudine (brand name: Epivir)
tenofovir disoproxil fumarate(brand name: Viread)
zidovudine (brand name: Retrovir)
Question: 258
Which of the following structure develops from dorsal mesentery?

A. Greater omentum
B. Lesser omentum
C. Liver
D. Diaphragm


Ans. A.
Explanation:
The portion of the dorsal mesentery that attaches to the greater curvature of the stomach, is known as the
dorsal mesogastrium. The part of the dorsal mesentery that suspends the colon is termed the mesocolon.
The dorsal mesogastrium develops into the greater omentum.
Question 259:
What is the structure seen in the given X-ray below?






A. Stent
B. Surgical clips
C. Foley catheter
D. Intravesical wire


Ans. A. Stent
Explanation:
Plain abdominal X ray showing a stent in the right and left ureter.
Ureteric stents,also known as double J stents or retrograde ureteric stents, is a urological catheter that
has two "J-shaped" (curled) ends, where one is anchored in the renal pelvis and the other inside the
bladder.
Stents are used for the free passage of urine from the kidney to the bladder, in adverse conditions such as
postoperative urologic procedures, and previously to lithotripsy and ureteral obstructions.
Indications
Stents may be uses for a short of long term period depending on the indication:
obstruction from urolithiasis
malignant obstruction (typically pelvic malignancies)
benign strictures
retroperitoneal fibrosis
bilateral ureteric stents




Question 260:
What is the diagnosis based on the following X-ray?


A. Uterine Fibroid
B. Bladder Carcinoma
C. Bladder stone
D. Renal Tuberculosis

Ans. C. Bladder stone
Explanation:
Multiple bladder calculi. Four oval radiopaque bladder stones are visible centrally in the pelvis. Most
bladder calculi are round or oval, but they may also be amorphous, laminated, or even spiculated.
Bladder stones are small mineral deposits that can form in the bladder. In most cases bladder stones
develop when the urine becomes very concentrated or when one is dehydrated.
This allows for minerals, such as calcium or magnesium salts, to crystallize and form stones.
In some cases bladder stones do not cause any symptoms and are discovered as an incidental finding on a
plain radiograph.
Bladder stones vary in their size, shape and texture- some are small, hard and smooth whereas others are
huge, spiked and very soft. One can have one or multiple stones. Bladder stones are somewhat more
common in men who have prostate enlargement. The large prostate presses on the urethra and makes it
difficult to pass urine. Over time, stagnant urine collects in the bladder and minerals like calcium start to
precipitate.
Radiography
The initial imaging study of choice is plain radiography of the kidneys, ureters, and bladder (KUB), which is
the least expensive and easiest radiologic test to obtain. Pure uric acid and ammonium urate stones are
radiolucent but may be coated with a layer of opaque calcium sediment. Laminations are common, with
the layers stratified according to metabolic and infectious status and the degree of periodic hematuria (see
the images below).






Multiple laminated bladder calculi in patient with neurogenic bladder.
Question 261:
By which method foreign DNA is introduced into a cell by a virus or viral vector?

A. Transduction
B. Transcription
C. Lysogenic conversion
D. Transformation


Ans. A. Transduction
Explanation:
Transduction is the process by which foreign DNA is introduced into a cell by a virus or viral vector. An
example is the viral transfer of DNA from one bacterium to another.
Question 262:
Which one of the following shows allosteric inhibition?

A. Malonic acid & succinate
B. 2,3 BPG
C. Amino acid alanine & pyruvate kinase
D. Citrate

Answer:B
Explanation:
Negative allosteric modulation (also known as allosteric inhibition) occurs when the binding of one ligand
decreases the affinity for substrate at other active sites. For example, when 2,3-BPG binds to an allosteric
site on hemoglobin, the affinity for oxygen of all subunits decreases.






Question 263:
Which of the following is seen in seropositive rheumatoid arthritis?
A. Multiple joints affected
B. Symmetrical joint symptoms
C. Joint pain and swelling
D. All


Answer:D
Explanation:
Positive for Rheumatoid factor in blood is seropositivity.
Patients with positive rheumatoid factor usually present with symptoms like
Joint deformities & disability
Symmetrical involvement of joints
Inflammation
Swelling and painful in multiple joints, especially of hands and feet.
Morning stiffness (short term)
Development of firm lumps near joints - "Rheumatoid nodules"
Deterioration of bone & cartilage (X- ray findings)
Question 264:
Which of the following is not seen in Anterior mediastinum

A. Thyroid tumour
B. Thymoma
C. Lymphoma
D. Neurogenic tumor


Answer:D
Explanation:



The anterior mediastinum is the portion of the mediastinum anterior to the pericardium and below
the thoracic plane.
It forms the anterior part of the inferior mediastinum
contains the thymus, lymph nodes, and may contain the portions of a retrosternal thyroid.
Mediastinal Tumors and Other Masses
Superior Mediastinum
Anterior Mediastinum
Posterior Mediastinum Middle Mediastinum
Lymphoma
Thymoma
Neurogenic tumors
Bronchogenic cyst
Thymoma
Teratoma
Lymphoma
Pericardial cyst
Thyroid lesions
Lymphoma
Gastroenteric hernia
Lymphoma
Metastatic carcinoma
Thyroid lesions
Parathyroid tumors
Parathyroid tumors
Question 265:
Struvite stone is caused by which metal?

A> Magnesium
B> Calcium
C> sodium & potassium
D> both (a) & (b)
Ans. Magnesium
Explanation:
Struvite, a crystalline substance is composed of magnesium ammonium phosphate (MgNH4PO4 ? 6H2O).
Struvite urinary stones have also been referred to as "infection stones" and "triple phosphate" stones.
Struvite stones can be caused by alkaline urine, steroid therapy, abnormal retention of urine, a urinary
tract infection, or another disorder of the urinary tract.
There are five primary types of commonly encountered urinary stones, i.e., calcium oxalate, calcium
phosphate, magnesium ammonium phosphate, uric acid, and cystine.


Question 266:
Which of the following statements about Graves disease is false?

A> Results in hyperthyroidism
B> Autoimmune disorder
C> Common in Male
D> Referred as Toxic diffuse goitre
Ans. Option C - Common in male
Explanation:
Graves' disease:
Autoimmune system disorder
Both men and women get affected;
Yet, 10 times more common in women than men
Affects younger women < 40 years
Results in overproduction of thyroid hormones (hyperthyroidism).
Signs and symptoms:
Anxiety
Irritability
Heat sensitivity
Increased perspiration/ warm and moist skin
Weight loss
Goiter (Glandular enlargement)
Menstrual cycle changes
Erectile dysfunction/ reduced libido
Graves Ophthalmopathy - Bulging eyes - Exophthalmos
Graves dermopathy - Thick, red skin on shins / top of feet.
Antibody for graves disease - Thyrotropin receptor antibody (TRAb) acts on the regulatory
pituitary hormone interfering the normal secretion of thyroxine.
TRAb overrides normal regulation causing an overproduction of thyroid hormones
(hyperthyroidism).
Question 267:
Aldosterone synthesis is stimulated by which of the following?

A.ACTH
B. Hyperkalemia
C.Hypernatremia
D.Exogenous steroids

Ans. B
Explanation:

Mineralocorticoid secretion is stimulated by hyperkalemia, angiotensin-H, ACTH and hyponatremia, in
reducing order of efficacy.
1. Aldosterone secretion in response to hyperkalemia is the most important and forms the basis for
renal regulation of body potassium balance.
2. Stimulation of aldosterone by angiotensin II (through renin-angioensin system) is important for the
correction of hypovolemia and hypotension in conditions like salt depletion or renal ischemia.
3. Stimulation of aldosterone secretion by ACTH results in diurnal variation of aldosterone secretion.
4. However, ACTH is not an important physiological regulator for aldosterone secretion.
Hyponatremia is a weak stimulator of aldosterone secretion
Question 268
Which of the following is false about Alzheimer's disease?

A. One in 10 people age 65 and older has Alzheimer's disease.
B. Alzheimer's disease is curable.

C. Cause dementia
D. All of the above

Ans. B
Explanation: Alzheimer's disease Also called: senile dementia. A progressive disease that destroys
memory and other important mental functions.
Memory loss and confusion are the main symptoms.
Currently, there is no cure for Alzheimer's. But drug and non-drug treatments may help with both
cognitive and behavioral symptoms.
The treatments available for Alzheimer's do not slow or stop the progression of the disease, but they may
help with the symptoms for a time.
There are three cholinesterase inhibitors to treat Alzheimer's:
Donepezil (Aricept)
Rivastigmine (Exelon)
Galantamine (Reminyl)

People may experience:
Cognitive: mental decline, difficulty thinking and understanding, confusion in the evening hours,
delusion, disorientation, forgetfulness, making things up, mental confusion, difficulty concentrating,
inability to create new memories, inability to do simple maths, or inability to recognise common
things
Behavioural: aggression, agitation, difficulty with self care, irritability, meaningless repetition of
own words, personality changes, restlessness, lack of restraint, or wandering and getting lost
Mood: anger, apathy, general discontent, loneliness, or mood swings
Psychological: depression, hallucination, or paranoia
Also common: behavioral symptoms, inability to combine muscle movements, jumbled speech, or
loss of appetite


Question 269
Which of the following is true about vitamin K?

A> anticoagulant
B> Prolong use of antimicrobial leads to deficiency
C> dietary allowance is 15-20 mg
D> all of the above

Ans. Prolong use of antimicrobial leads to deficiency
Explanation:
Certain people are at increased risk if they:
take coumarin anticoagulants such as warfarin, which thins the blood
are taking antibiotics
have a condition that causes the body to not absorb fat properly (fat malabsorption)
have a diet that is extremely lacking in vitamin K
Vitamin K is a group of structurally similar, fat-soluble vitamins the human body requires for complete
synthesis of certain proteins that are prerequisites for blood coagulation and which the body also needs for
controlling binding of calcium in bones and other tissues
The body needs vitamin K to produce prothrombin, a protein and clotting factor that is important in blood
clotting and bone metabolism.
Without vitamin K, blood coagulation is seriously impaired, and uncontrolled bleeding occurs. Preliminary
clinical research indicates that deficiency of vitamin K may weaken bones, potentially leading to
osteoporosis, and may promote calcification of arteries and other soft tissues
Dietary allowance for adults per day- 50-100 mg.
Question 270
Which drugs needs continuous monitoring of prothrombin time?

A.Aspirin
B.Lepirudin
C.Digoxin
D. Coumadin

Ans.D
Explanation: Coumadin (warfarin) is an anticoagulant.
Warfarin is a coumarin anticoagulant used for the prophylaxis and treatment of thromboembolic
complications associated with cardiac valve replacement and atrial fibrillation,as well as the prophylaxis
and treatment of venous thrombosis and pulmonary embolism.Increased metabolism of warfarin results in
insufficient prolongation of prothrombin time.

Question 271
Which of the following are the risk factor for cutaneous lymphoma?
A> Age
B> Gender
C> Weakened immune system
D> All

Answer:D
Explanation:
Risk Factors for Lymphoma of the Skin
Age Age is an important risk factor for this disease, with most cases occurring in people in their 50s and
60s. But some types of skin lymphoma can appear in younger people, even in children.
Gender and race Most (but not all) types of skin lymphoma are more common in men than in women.
Most also tend to be more common in African-Americans than in whites. The reasons for this are not
known.
Weakened immune systemSkin lymphomas may be more common in people with acquired
immunodeficiency syndrome (AIDS), who have a weakened immune system. They may also be more
common in people who have had an organ transplant such as a heart, kidney or liver transplant. These
people must take drugs that suppress their immune system, which may raise the risk of skin lymphoma (or
lymphomas in other parts of the body).
Infections Infection with the human immunodeficiency virus (HIV), the virus that causes AIDS, may
increase a person's risk of skin lymphoma.

Question 272:
Which is not included in AIDS related complex?
A. Ectopic pregnancy
B. Recurrent genital candidiasis
C. Generalised lymphadenopathy
D. Chronic diarrhea

Answer- A
Explanation: HIV symptoms: AIDS related complex (ARC)
It belongs to class B of HIV symptoms. The patients at this stage have various diseases that occur because
the HI virus has weakened the immune system.
The following HIV signs may have patients with ARC:
Long-lasting diarrhea (over four weeks)
Unintended heavy weight loss
Long lasting fever

Night sweats
Bacterial infections caused by bacteria
Bacterial blood poisoning (sepsis)
Phthisis
Herpes zoster
Oral hairy leukoplakia (whitish changes on the lateral tongue border)
Fungi caused by fungi
HIV symptoms ? Women: vaginal inflammation caused by fungi, malignant changes in the cervix

Question 273:
Which is the treatment of choice for irradiation in Chordoma?

A> Protons
B> Electrons
C> Gamma radiation
D> 3D - CRT

Answer: Option A - Proton Therapy
Explanation:
Chordoma:
Slow-growing neoplasm
Arising from cellular remnants of notochord.
Arise from bone in skull base and along spinal cord.
Most common locations -
Cranially at clivus
In sacrum at bottom of spine
Radiation therapy:
Are relatively radioresistant
High doses of radiation required to control.
Hence, high focus radiation like proton therapy and carbon ion therapy are preferred than
conventional radiation methods.
Close proximity to vital structures like brain stem, requires high precision and accuracy for
any planned surgical resection.
Radiation with high accuracy and minimal damage with maximal safety is delivered.

Question 274:
A woman shows symptoms of massive pulmonary thromboembolism.The gross appearance of liver
autopsy is shown. Which of the following statement best characterizes the patient's condition?







A> Metastasis from PE

B> Angiosarcoma
C> Colonic adenocarcinoma with metastasis
D>Locally invaded hepatocellular carcinoma

Answer: Option C
Explanation:
The figure shows appearance of metastatic lesions from a malignant neoplasm with multiple tumor masses
The liver is the most common site of metastases for tumor sites that drain initially via the portal
circulation. Metastatic liver disease is found in 10% to 25% of patients having surgery for primary
colorectal cancer
Surgical resection is the most effective therapy for metastatic colorectal cancer isolated to the liver.
Question 275
Which of the following statement is false about MR vaccination campaign launched by WHO?

A>Children from 9 months to less than 15 vaccinated
B>Congenital rubella syndrome (CRS), responsible for irreversible birth defects
C>India has not yet launched this campaign
D>Will replace routine immunization for measles vaccine

Answer:C
Explanation
One of the world's largest vaccination campaign against measles, a major childhood killer disease,
and congenital rubella syndrome (CRS), responsible for irreversible birth defects.
India, along with ten other WHO South East Asia Region member countries, have resolved to
eliminate measles and control rubella/congenital rubella syndrome (CRS) by 2020
.All children from 9 months to less than 15 years of age will be given a single shot of Measles-
Rubella (MR) vaccination during the campaign
. Following the campaign, MR vaccine will become a part of routine immunization and will replace
measles vaccine, currently given at 9-12 months and 16-24 months of age of child.
For those children who have already received such vaccination, the campaign dose would provide
additional boosting to them.

Question 276.
Which of the following true regarding Hemophilia A

A>Serum levels of factor VIII are decreased.
B>Deficiency of factor IX
C>PT increased
D>FIT decreased

Answer: A
Explanation
Hemophilia is an X linked disorder of coagulation caused by the deficiency in a circulating plasma
protein. Hemophilia A is caused by the deficiency of factor VIII, and hemophilia B is caused by the
deficiency of factor IX.
It is PTT which is affected (increased) and not PT (unaffected).
Factor VIII is involved in the intrinsic pathway which is measured by PTT and not in extrinsic
pathway which is measured as PT.
Bleeding is the common manifestation of hemophilia and the common bleeding manifestations are
hemarthoses, hematomas, mucocutaneous bleeding, intracranial bleeding, hematuria and
pseudotumor.
Question 277
Marked bleeding is seen in which of following conditions?

A>VMA disease
B>Haemophilia A
C>Haemophilia B
D>ALL
Answer: D
Explanation
Bleeding disorders can be inherited or acquired. Inherited disorders are passed down through genetics.
Acquired disorders can develop or spontaneously occur later in life. Some bleeding disorders can result in
severe bleeding following an accident or injury. In other disorders, heavy bleeding can happen suddenly
and for no reason.
There are numerous different bleeding disorders, but the following are the most common ones:
Hemophilia A and B are conditions that occur when there are low levels of clotting factors in your blood. It
causes heavy or unusual bleeding into the joints. Though hemophilia is rare, it can have life-threatening
complications.
Factor II, V, VII, X, or XII deficiencies are bleeding disorders related to blood clotting problems or abnormal
bleeding problems.


von Willebrand's disease is the most common inherited bleeding disorder. It develops when the blood
lacks von Willebrand factor, which helps the blood to clot.
Question 278
Reed sternberg cells are found in

A>Hodgkin's disease
B>Sickle cell anaemia
C> Thalassemia
D>CML

Answer: A
Explanation:
Reed-Sternberg cells
They are usually derived from B lymphocytes, classically considered crippled germinal center B cells.
Seen against a sea of B cells, they give the tissue a moth-eaten appearance.
They are named after Dorothy Reed Mendenhall and Carl Sternberg, who provided the first
definitive microscopic descriptions of Hodgkin's disease.
Reed-Sternberg cells are large and are either multinucleated or have a bibbed nucleus (thus
resembling an "owl's eye" appearance) with prominent eosinophilic inclusion-like nucleoli.
Reed-Sternberg cells are CD30 and CD15 positive, usually negative for CD20 and CD45.
The presence of these cells is necessary in the diagnosis of Hodgkin's lymphoma - the absence of
Reed-Sternberg cells has very high negative predictive value.
They can also be found in reactive lymphadenopathy (such as infectious mononucleosis,
carbamazepine associated lymphadenopathy) and very often in other types of non-Hodgkin
lymphomas.
A special type of Reed-Sternberg cells is the lacunar histiocyte, whose cytoplasm retracts when
fixed in formalin, so the nuclei give the appearance of cells that lie with empty spaces (called
lacunae) between them.
These are characteristic of the nodular sclerosis subtype of Hodgkin's lymphoma
Question 279
A bedridden patient experiences acute chest pain which is worsened by breathing.Which imaging
techniques could be helpful?

A>USG
B>X-ray chest
C>Ventilation perfusion scan
D>CT scan
Answer:
Explanation. Symptoms of pulmonary embolism (PE) are
Sudden-onset dyspnea
Tachypnea



Chest pain of a "pleuritic" nature (worsened by breathing)
Cough
Hemoptysis
More severe cases can include signs such as cyanosis, collapse, and circulatory instability.
On physical examination, a pleural rub may be audible by stethoscope over affected areas of the lung.
Strain on the right ventricle may be detected as a left parasternal heave, a loud pulmonary component of
the second heart sound, raised jugular venous pressure, and more rarely leg swelling.
The gold standard for diagnosing pulmonary embolism (PE) is pulmonary angiography. Pulmonary
angiography is used less often due to wider acceptance of CT scans, which are non-invasive.
Question 280:
Chordoma arises from:
A> Pharyngeal bursa

B> Notochord
C> Rathke's pouch
D>Luschka's bursa

Answer: Option - B
Explanation: Chordoma is a rare slow-growing neoplasm thought to arise from cellular remnants of the
notochord.
Chordomas can arise from bone in the skull base and anywhere along the spine. The two most common
locations are cranially at the clivus and in the sacrum at the bottom of the spine.
There are three histological variants of chordoma: classical (or "conventional"), chondroid and
dedifferentiated.


Question 281:

What is the effect of Progesterone only pills?

A> Completely suppresses ovulation
B> Thin lining of uterus
C> Thick cervical mucus
D> All of the above


Answer: Option D - All of the above

Explanation:

Progestogen-only pills / Progestin-only Pills (POP) / Morning after pills -
Contraceptive pills
Contain only synthetic progestins & doesnt contain estrogen.
Mechanism:
Mucus thickening in neck of womb:
Penetration of sperm to reach egg and womb becomes difficult.
Prevents ovulation:
Lining of the uterus becomes thin
Fertilized egg implantation is prevented
Advantages:
Doesn't interfere with breastfeeding
Also helps in premenstrual symptoms and painful periods

Question 282:

Which metal results in "Saturnine gout" formation?
A> Cadmium

B> Lead
C> Beryllium
D> Mercury

Answer: Option - B - Lead
Explanation:
One manifestation of chronic lead toxicity is the rheumatologic entity known as saturnine gout.
Illicitly distilled beverages may inadvertently contain harmful toxins, like metallic lead.
Lead has been known to play a role in purine metabolism & renal insufficiency

Question 283:

Which drug decreases the bone resorption in osteoporosis?

A>Teriparatide
B> Risedronate
C> Cortisone
D> Cimetidine

Answer: Option B - Risedronate
Explanation: Risedronate bisphosphonates inhibits bone resorption by actions on osteoclast precursors in
osteoporosis patients
Risedronate:
Aminobisphosphonate
Indications:

Prevention & treatment of osteoporosis
Mechanism of action:
Inhibits bone resorption by action on osteoclasts
Reduce bone remodelling
More potent in blocking the bone dissolution process.
Teriparatide, an PTH analog, recombinant human PTH is also used, yet in severe cases of
osteoporosis, improving the skeletal microarchitecture
Question 284
Phenylketonuria is due to deficiency of:

A> Phenylalanine
B>Phenylalanine hydroxylase ( PAH)
C> Phenylene
D> All of these
Answer B
Explanation: A birth defect that causes an amino acid called phenylalanine to build up in the body.
PKU is an autosomal recessive metabolic genetic disorder.
PKU is characterized by homozygous or compound heterozygousmutations in the gene for the hepatic
enzyme phenylalanine hydroxylase (PAH), rendering it nonfunctional.
This enzyme is necessary to metabolize the amino acid phenylalanine (Phe) to the amino acid tyrosine
(Tyr). When PAH activity is reduced, phenylalanine accumulates and is converted into phenylpyruvate (also
known as phenylketone), which can be detected in the urine.
The PAH gene is located on chromosome 12 in the bands 12q22-q24.1. More than 400 disease-causing
mutations have been found in the PAH gene.
Question 285
WHICH IS NOT CORRECT:
A> MRI needed to access haemorrhage
B> GCS assessment helps in prognosis
C> Haematoma must be operated
D> all of the above

Answer- C
Explanation - Treatment of hematoma depends on the location, symptoms, and the clinical situation. Some
may require no treatment at all while others may be deemed a medical emergency.
Simple therapies at home may be utilized in treating superficial (under the skin) hematomas. Most injuries

and bruises can be treated with resting, icing, compression, and elevating the area. This is remembered by
the acronym RICE.
These measures usually help to reduce inflammation and diminish its symptoms.
Rest
Ice (Apply the ice or cold pack for 20 minutes at a time, 4 to 8 times a day.)
Compress (Compression can be achieved by using elastic bandages.)
Elevate (Elevation of the injured area above the level of the heart is recommended.)
Medical treatment for a hematoma
For certain small and symptom-free hematomas no medical treatment may be necessary. On the other
hand, symptomatic hematomas or those located in certain locations sometimes require medical or surgical
treatment.
Even though no specific mediation is available for the treatment of hematomas, management of any
related symptoms can be achieved by medications. For example, pain from a hematoma can be treated
with pain medications such as acetaminophen (Tylenol).
Surgical drainage is a common method of treatment for certain hematomas.
Question 286
Maastricht classification of donation after cardiac death.What category is stage 3 ?
A>Awaiting cardiac arrest
B>Brought in dead
C>Unsuccessful resuscitation
D>Cardiac arrest after brain-stem death
Answer : A>Awaiting cardiac arrest
Non heart beating donors are grouped according to Maastricht classification .

Question 287
What is the cause of myocardial shock other than MI ?

A> acute mitral regurgitation
B>ventricular septal rupture
C> isolated right ventricular shock
D>all of the above
Answer : D> All of the above
Explanation:
Left ventricular dysfunction (LVD) - Most frequent cause of cardiogenic shock

Followed by,
Acute mitral valve regurgitation

Ventricular septal defect
Isolated right ventricular shock
Tamponade/ cardiac rupture
Question 288
Nitric oxide acts by increasing ?

A> BRCA 1
B> BRCA 2
C>Interleukin
D>cGMP
Answer: D> cGMP
Explanation:
Nitric oxide diffuses to the surrounding smooth muscle cells, increasing cGMP .
Cyclic guanosine monophosphate (cGMP)
Cyclic nucleotide derived from guanosine triphosphate (GTP).
Function:
cGMP acts as a second messenger much like cyclic AMP.
Mechanism of action:
Activation of intracellular protein kinases in response to the binding of membrane-
impermeable peptide hormones.

Question 289:
Positive acid schiff macrophages seen in ?

a.>Whipples disease
B.> Crohns disease
C.> AIDS
D> None of the above

Answer : A> Whipples disease.
Explanation : The traditional laboratory diagnosis is based on light microscopy, which shows diastase-
resistant, periodic acid-Schiff (PAS)-positive, non-acid-fast granules in ..... The distinction could be made by
acid-fast staining, which is positive for patients infected with M. avium and negative for those with
Whipple's disease.
Question 290:
A boy presented with multiple non suppurative osteomyelitis with sickle cell anaemia. What will be the
causative organism?

A. Salmonella
B. S. aureus
C. H. influenzae


D. Enterobacter species

Ans. A. Salmonella
Explanation:
Following are the various micro -organism involved in osteomyelitis
Age group
Most common organisms
Newborns (younger than 4 mo) S. aureus, Enterobacter species, and group A and B Streptococcus
species
Children (aged 4 mo to 4 y)
S. aureus, group A Streptococcus species, Haemophilus influenzae, and
Enterobacter species
Children, adolescents (aged 4 y S. aureus (80%), group A Streptococcus species, H. influenzae, and
to adult)
Enterobacter species
Adult
S. aureus and occasionally Enterobacter or Streptococcus species
Sickle cell anemia patients
Salmonella species are most common in patients with sickle cell disease
Question 291:
Term pathology refers to:
A. Work
B. Function
C. Details
D. Explains


Ans. A. Work
Explanation:
Pathology is the medical term for the way a disease works.
The science of the causes and effects of diseases, especially the branch of medicine that deals with the
laboratory examination of samples of body tissue for diagnostic or forensic purposes.
Question 292:
1 yr child weighing 6 kg is suffering from Acute Gastroenteritis along with signs of sunken eyes & skin
pinch going back to normal very rapidly. What will be your management?

A. RL infusion 120 ml in the first hour followed by 360 ml in the next 5 hours
B. RL INFUSION 180 ml in the first hour followed by 420 ml in the next 5 hours
C. RL INFUSION 180 ML IN the first hour followed by 480 ml in the next 5 hours
D. RL INFUSION 240 ml in the first hour followed by 360 ml in the next 5 hours


Ans. B.RL INFUSION 180 ml in the first hour followed by 420 ml in the next 5 hours
Explanation:

Severe dehydration constitutes a medical emergency requiring immediate resuscitation with
intravenous fluids.
Intravenous access should be obtained, and patients should be administered a bolus of 20-30
mL/kg lactated Ringer's (LR) or normal saline (NS).
If pulse, perfusion, and/or mental status do not improve, a second bolus should be administered.
After this, the patient should be given an infusion of 70 mL/kg LR or NS over 5 hours (children < 12
months) or 2.5 hours (older children).
If no peripheral veins are available, an intraosseous line should be placed. Serum electrolytes,
bicarbonate, urea/creatinine, and glucose levels should be sent.
Question 293:
What constitutes malpighian layer?

A. Corneum lucidum
B. Corneum spinosum
C. Spinosum and basale
D. Basale granulosum


Ans.C.Spinosum and basale
Explanation:
The Malpighian layer of the skin is generally defined as both the stratum basale and stratum spinosum as a
unit, although it is occasionally defined as the stratum basale specifically,or the stratum spinosum
specifically. It is named after Marcello Malpighi.
Question 294:
Mechanism of action colchicine in acute gout
A. Uric acid nephrolithiasis.
B. Deficiency of enzyme Xanthine oxidase.
C. Increase in serum urate concentration.
D. Renal disease involving interstitial tissues.


Ans.B.Deficiency of enzyme Xanthine oxidase.
Explanation:
Gout is a hereditary disorder with increase in serum uric acid due to increased production, or
decreased excretion of uric acid and uric salt.
Thought to be caused by lack of an enzyme needed to completely metabolise purines for renal
excretion.











Question 295:
Oxygen therapy may not be useful in

A. Asthma
B. Pneumonia
C. Subglottic stenosis
D. Pulmonary fibrosis


Ans: D.Pulmonary fibrosis
Explanation:
Many EMS protocols indicate that oxygen should not be withheld from any patient, while other protocols
are more specific or circumspect. However, there are certain situations in which oxygen therapy is known
to have a negative impact on a patient's condition like paraquat poisoning,pulmonary fibrosis and lung
damage resulting from bleomycin treatment.
Question 296:
New born baby with heart rate less than 60 beats per minute can be resuscitated by all except
A: chest compression
B: oxygen therapy
C: tactile stimulation
D: slapping the back

Ans: D.slapping the back










Explanation:
The first three options are indicated while slapping the back is not recommended in a newborn who has
Heart rate less than 60 beats per minute.
Question 297:
Mobitz type 2 second degree AV block is seen in all except:

A: Hypothyroidism
B: Coronary Artery Disease
C: Sarcoidosis
D: Cushing syndrome

Ans: D. Cushing syndrome
Explanation :
Delay or lack of conduction through the atrioventricular (AV) node and below has multiple causes.
Degenerative changes (eg, fibrosis, calcification, or infiltration) are the most common cause of non-
ischeamic AV block. Idiopathic fibrosis or calcification of the AV conduction system, commonly seen in the
elderly, can cause complete AV block.
Causes of Mobitz type 2 second degree AV block are -
Damage of the conduction system from coronary artery disease, valve surgery, myocardial infarction,
myocarditis, infiltrative cardiomyopathies (sarcoidosis, hemochromatosis), myxedema, Lyme disease,
neuromuscular disease, and AV junction ablation [6]













Question 298:
When can one diagnose acute respiratory distress in a child ?

A: Within 7 days of known clinical insult
B: Respiratory failure not fully explained
C: Left ventricular dysfunction
D: All of the above

Ans: D.
Explanation
Question 299:
A 6-year-old boy experienced life threatening shock ,his CT scan showed large amount of ascites, bowel
wall thickening and poor or absent enhancement of the strangulated bowel segment, showing
gangrenous bowel on surgical exploration.
True about anastomosis is-

A:Should be done by continuous layers as it takes less time
B: Should be Done with catgut
C: Should be Done by single layer seromuscular lembert sutures
D: Should be Done by Single layer taking submucosa


Ans C
Explanation:
Diagnosis is of congenital IH with strangulated small bowel with gangrenous small bowel
Transmesenteric hernia was the most common type in older children as well as in neonates .
IH results from incomplete closure of surgically created mesenteric defects, and usually acquired
resulting from previous abdominal surgery especially Roux-en-Y anastomosis
Anastomosis should be done by single layer seromuscular lembert sutures
The Lambert suture generally is used in abdominal surgery. It is an inverting suture, that can be
either continuous or interrupted, used to join two segments of an intestine without entering the
lumen (the inner channel through which stomach contents flow).
Question 300
In ACLS which drug can be given following ventricular fibrillation after cardiac arrest other than
epinephrine?

A. Amiodarone
B. Dopamine
C. Adenosine
D. Atropine


Ans. A. Amiodarone
Explanation:
V-Fib or VF is the most common rhythm that occurs immediately after cardiac arrest. In this rhythm, the
heart beats with rapid, erratic electrical impulses.
Treatment:
Shock / Defibrillation: every 2 minutes in a single one shock, successive, shockable increments
200 joules - Followed by immediate CPR for 2 minutes / give and circulate a drug(s)
300 joules - Followed by immediate CPR for 2 minutes / give and circulate a drug(s)
360 joules - Followed by immediate CPR for 2 minutes / give and circulate a drug(s)
Drugs :
Give Epinephrine 1mg of a 1:10,000 solu,on every 3 to 5 minutes [No Limit]
Give either:
Amiodarone [if not contraindicated, can be given 2x]: 300mg first dose / 150mg second
dose at 3 to 5 minutes increments.
Lidocaine: First dose: 1mg/kg or 1.5 mg/kg. Can repeat it at half the original dose up to a total of 3 mg/kg
[Second and remaining doses are given at either 0.5mg/kg or 0.75mg/kg depending on your star,ng
dosage.]



This post was last modified on 30 July 2021